GRE阅读真题大全

合集下载

GRE国内阅读理解真题

GRE国内阅读理解真题

1994年10月SECTION AThe Fourteenth Amendment to the United States Constitution, ratified in 1868, prohibits state governments from denying citizens the “equal protection of the laws.”Although precisely what the framers of the amendment meant by this equal protection clause remains unclear, all interpreters agree that the framers’ immediate objective was to provide a constitutional warrant for the Civil Rights Act of 1866, which guaranteed the citizenship of all persons born in the United States and subject to United States jurisdiction. This declaration, which was echoed in the text of the Fourteenth Amendment, was designed primarily to counter the Supreme Court’s ruling in Dred Scott v. Sandford that Black people in the United States could be denied citizenship. The act was vetoed by President Andrew Johnson, who argued that the Thirteenth Amendment, which abolished slavery, did not provide Congress with the authority to extend citizenship and equal protection to the freed slaves. Although Congress promptly overrode Johnson’s veto, supporters of the act sought to ensure its constitutional foundations with the passage of the Fourteenth Amendment.The broad language of the amendment strongly suggests that its framers were proposing to write into the Constitution not a laundry list of specific civil rights but a principle of equal citizenship that forbids organized society from treating any individual as a member of an inferior class. Yet for the first eight decades of the amendment’s existence, the Supreme Court’s interpretation of the amendment betrayed this ideal of equality. In the Civil Rights Cases of 1883, for example, the Court invented the “state action” limitation, which asserts that “private” decisions by owners of public accommodations and other commercial businesses to segregate their facilities are insulated from the reach of the Fourteenth Amendment’s guarantee of equal protection under the law.After the Second World War, a judicial climate more hospitable to equal protection claims culminated in the Supreme Court’s ruling in Brown v. Board of Education that racially segregated schools violated the equal protection clause of the Fourteenth Amendment. Two doctrines embraced by the Supreme Court during this period extended the amendment’s reach. First, the Court required especially strict scrutiny of legislation that employed a “suspect classification,” meaning discrimination against a group on grounds that could be construed as racial. This doctrine has broadened the application of the Fourteenth Amendment to other, nonracial forms of discrimination, for while some justices have refused to find any legislative classification other than race to be constitutionally disfavored, most have been receptive to arguments that at least some nonracial discriminations, sexual discrimination in particular, are “suspect” and deserve this heightened scrutiny by the courts. Second, the Court relaxed the state action limitation on the Fourteenth Amendment, bringing new forms of private conduct within the amendment’s reach.17. Which of the following best describes the main idea of the passage?(A) By presenting a list of specific rights, framers of the Fourteenth Amendmentwere attempting to provide a constitutional basis for broad judicial protection ofthe principle of equal citizenship.(B) Only after the Supreme Court adopted the suspect classification approach toreviewing potentially discriminatory legislation was the applicability of theFourteenth Amendment extended to include sexual discrimination.(C) Not until after the Second World War did the Supreme Court begin to interpretthe Fourteenth Amendment in a manner consistent with the principle of equalcitizenship that it expresses.(D) Interpreters of the Fourteenth Amendment have yet to reach consensus withregard to what its framers meant by the equal protection clause.(E) Although the reluctance of judges to extend the reach of the FourteenthAmendment to nonracial discrimination has betrayed the principle of equalcitizenship, the Supreme Court’s use of the state action limitation to insulateprivate activity from the amendment’s reach has been more harmful.18. The passage suggests that the principal effect of the state action limitation was to(A) allow some discriminatory practices to continue unimpeded by the FourteenthAmendment(B) influence the Supreme Court’s ruling in Brown v, Board of Education(C) provide expanded guidelines describing prohibited actions(D) prohibit states from enacting laws that violated the intent of the Civil Rights Actof 1866(E) shift to state governments the responsibility for enforcement of laws prohibitingdiscriminatory practices19. The author’s position regarding the intent of the framers of the FourteenthAmendment would be most seriously undermined if which of the following were true?(A) The framers had anticipated state action limitations as they are described in thepassage.(B) The framers had merely sought to prevent discriminatory acts by federalofficials.(C) The framers were concerned that the Civil Rights Act of 1866 would beoverturned by the Supreme Court.(D) The framers were aware that the phrase “equal protection of the laws” hadbroad implications.(E) The framers believed that racial as well as non-racial forms of discriminationwere unacceptable.20. According to the passage, the original proponents of the Fourteenth Amendmentwere primarily concerned with(A) detailing the rights afforded by the principle of equal citizenship(B) providing support in the Constitution for equal protection for all citizens of theUnited States(C) closing a loophole that could be used to deny individuals the right to sue forenforcement of their civil rights(D) asserting that the civil rights protected by the Constitution included nonracialdiscrimination as well as racial discrimination(E) granting state governments broader discretion in interpreting the Civil RightsAct of 186621. The author implies that the Fourteenth Amendment might not have been enacted if(A) Congress’ authority with regard to legislating civil rights had not beenchallenged(B) the framers had anticipated the Supreme Court’s ruling in Brown v. Board ofEducation(C) the framers had believed that it would be used in deciding cases ofdiscrimination involving non-racial groups(D) most state governments had been willing to protect citizens’ civil rights(E) its essential elements had not been implicit in the Thirteenth Amendment22. According to the passage, which of the following most accurately indicates thesequence of the events listed below?I. Civil Rights Act of 1866II. Dred Scott v. SandfordIII. Fourteenth AmendmentIV. Veto by President Johnson(A) I, II, III, IV(B) I, IV, II, III(C) I, IV, III, II(D) II, I, IV, III(E) III, II, I, IV23. Which of the following can be inferred about the second of the two doctrines referredto in lines 39-41 of the passage?(A) It caused some justices to rule that all types of discrimination are prohibited bythe Constitution.(B) It shifted the focus of the Supreme Court from racial to nonracial discrimination.(C) It narrowed the concern of the Supreme Court to legislation that employed asuspect classification.(D) It caused legislators who were writing new legislation to reject language thatcould be construed as permitting racial discrimination.(E) It made it more difficult for commercial businesses to practice racialdiscrimination.The Earth’s magnetic field is generated as the molten iron of the Earth’s outer core revolves around its solid inner core. When surges in the molten iron occur, magnetic tempests are created. At the Earth’s surface, these tempests can be detected by changes in the strength of the Earth’s magnetic field. For reasons not fully understood, the field itself reverses periodically every million years or so. During the past million years, for instance, the magnetic north pole has migrated between the Antarctic and the Arctic.Clearly, geophysicists who seek to explain and forecast changes in the field must understand what happens in the outer core. Unlike meteorologists, however, they cannot rely on observations made in their own lifetimes. Whereas atmospheric storms arise in a matter of hours and last for days, magnetic tempests develop over decades and persist for centuries. Fortunately scientists have been recording changes in the Earth’s magnetic field for more than 300 years.24. In the passage, the author is primarily concerned with(A) analyzing a complicated scientific phenomenon and its impact on the Earth’ssurface features(B) describing a natural phenomenon and the challenges its study presents toresearchers(C) discussing a scientific field of research and the gaps in researchers’methodological approaches to it(D) comparing two distinct fields of physical science and the different researchmethods employed in each(E) proposing an explanation for a geophysical phenomenon and an experiment thatcould help confirm that explanation25. The passage suggests which of the following about surges in the Earth’s outer core?(A) They occur cyclically every few decades.(B) They can be predicted by changes in the Earth’s inner core.(C) They are detected through indirect means.(D) They are linked to disturbances in the Earth’s atmosphere.(E) They last for periods of about 1 million years.26. It can be inferred from the passage that geophysicists seeking to explain magnetictempests ought to conduct research on the Earth’s outer core because the Earth’souter core(A) is more fully understood than the Earth’s magnetic field(B) is more easily observed than the Earth’s magnetic field(C) has been the subject of extensive scientific observation for 300 years(D) is involved in generating the Earth’s magnetic field(E) reflects changes in the inner core caused by magnetic tempests27. In the second paragraph, the author is primarily concerned with(A) stating a limitation that helps determine a research methodology(B) making a comparative analysis of two different research methodologies(C) assessing the amount of empirical data in the field of physical science(D) suggesting an optimistic way of viewing a widely feared phenomenon(E) describing a fundamental issue and discussing its future impact on societySECTION BThe defoliation of millions of acres of trees by massive infestations of gypsy moth caterpillars is a recurring phenomenon in the northeastern United States. In studying these outbreaks, scientists have discovered that affected trees fight back by releasing toxic chemicals, mainly phenols, into their foliage. These noxious substances limit caterpillars’growth and reduce the number of eggs that female moths lay. Phenols also make the eggs smaller, which reduces the growth of the following year’s caterpillars. Because the number of eggs a female moth produces is directly related to her size, and because her size is determined entirely by her feeding success as a caterpillar, the trees’ defensive mechanism has an impact on moth fecundity.The gypsy moth is also subject to attack by the nucleopolyhedrosis virus, or wilt disease, a particularly important killer of the caterpillars in outbreak years. Caterpillars contract wilt disease when they eat a leaf to which the virus, encased in a protein globule, has become attached. Once ingested by a caterpillar, the protein globule dissolves, releasing thousands of viruses, or virions, that after about two weeks multiply enough to fill the entire body cavity. When the caterpillar dies, the virions are released to the outside, encased in a new protein globule synthesized from the caterpillar’s tissues and ready to be picked up by other caterpillars.Knowing that phenols, including tannins, often act by associating with and altering the activity of proteins, researchers focused on the effects on caterpillars of ingesting the virus and leaves together. They found that on tannin-rich oak leaves, the virus is considerably less effective at killing caterpillars than when it is on aspen leaves, which are lower in phenols. In general, the more concentrated the phenols in tree leaves, the less deadly the virus. Thus, while highly concentrated phenols in tree leaves reduce the caterpillar population by limiting the size of caterpillars and, consequently, the size of the female’s egg cluster, these same chemicals also help caterpillars survive by disabling the wilt virus. Forest stands of red oaks, with their tannin-rich foliage, may even provide caterpillars with safe havens from disease. In stands dominated by trees such as aspen, however, incipient gypsy moth outbreaks are quickly suppressed by viral epidemics.Further research has shown that caterpillars become virtually immune to the wilt virus as the trees on which they feed respond to increasing defoliation. The trees’ own defenses raise the threshold of caterpillar vulnerability to the disease, allowing populations to grow denser without becoming more susceptible to infection. For these reasons, the benefits to the caterpillars of ingesting phenols appear to outweigh the costs. Given the presence of the virus, the trees’ defensive tactic apparently has backfired.17. Which of the following statements best expresses the main point of the passage?(A) Recurring outbreaks of infestation by gypsy moth caterpillars have had adevastating impact on trees in the northeastern United States.(B) A mechanism used by trees to combat the threat from gypsy moth caterpillarshas actually made some trees more vulnerable to that threat.(C) Although deadly to gypsy moth caterpillars, wilt disease has failed tosignificantly affect the population density of the caterpillars.(D) The tree species with the highest levels of phenols in their foliage are the mostsuccessful in defending themselves against gypsy moth caterpillars.(E) In their efforts to develop new methods for controlling gypsy moth caterpillars,researchers have focused on the effects of phenols in tree leaves on the insects’growth and reproduction.18. In lines 12-14, the phrase “the trees’ defensive mechanism has an impact on mothfecundity” refers to which of the following phenomena?(A) Female moths that ingest phenols are more susceptible to wilt virus, whichcauses them to lay smaller eggs.(B) Highly concentrated phenols in tree leaves limit caterpillars’ food supply,thereby reducing the gypsy moth population.(C) Phenols attack the protein globule that protects moth egg clusters, making themvulnerable to wilt virus and lowering their survival rate.(D) Phenols in oak leaves drive gypsy moths into forest stands dominated by aspens,where they succumb to viral epidemics.(E) The consumption of phenols by caterpillars results in undersized female gypsymoths, which tend to produce small egg clusters.19. It can be inferred from the passage that wilt disease virions depend for their survivalon(A) protein synthesized from the tissues of a host caterpillar(B) aspen leaves with high concentrations of phenols(C) tannin-rich oak leaves(D) nutrients that they synthesize from gypsy moth egg clusters(E) a rising threshold of caterpillar vulnerability to wilt disease20. Which of the following, if true, would most clearly demonstrate the operation of thetrees’ defensive mechanism as it is described in the first paragraph of the passage?(A) Caterpillars feeding on red oaks that were more than 50 percent defoliated grewto be only two-thirds the size of those feeding on trees with relatively intactfoliage.(B) Oak leaves in areas unaffected by gypsy moths were found to have higher levelsof tannin on average than aspen leaves in areas infested with gypsy moths.(C) The survival rate of gypsy moth caterpillars exposed to the wilt virus was 40percent higher for those that fed on aspen leaves than for those that ate oakleaves.(D) Female gypsy moths produced an average of 25 percent fewer eggs in areaswhere the wilt virus flourished than did moths in areas that were free of thevirus.(E) Gypsy moth egg clusters deposited on oak trees were found to have relativelylarge individual eggs compared to those deposited on aspen trees.21. Which of the following best describes the function of the third paragraph of thepassage?(A) It resolves a contradiction between the ideas presented in the first and secondparagraphs.(B) It introduces research data to support the theory outlined in the secondparagraph.(C) It draws a conclusion from conflicting evidence presented in the first twoparagraphs.(D) It shows how phenomena described in the first and second paragraphs act incombination.(E) It elaborates on the thesis introduced in the first paragraph after a digression inthe second paragraph.22. It can be inferred from the passage that gypsy moth caterpillars become immune tothe wilt virus as a result of(A) consuming a wide range of nutrients from a variety of leaf types(B) feeding on leaves that contain high levels of phenols(C) producing fewer offspring, which favors the survival of the hardiest individuals(D) ingesting the virus together with leaves that do not contain tannin(E) growing population density, which outstrips the ability of the virus to multiplyand spread23. Which of the following statements about gypsy moth caterpillars is supported byinformation presented in the passage?(A) Wilt disease is more likely to strike small gypsy moth caterpillars than largeones.(B) The concentration of phenols in tree leaves increases as the gypsy mothcaterpillar population dies off.(C) Female gypsy moth caterpillars stop growing after they ingest leaves containingphenols.(D) Differing concentrations of phenols in leaves have differing effects on the abilityof the wilt virus to kill gypsy moth caterpillars.(E) The longer a gypsy moth population is exposed to wilt disease, the greater thelikelihood that the gypsy moth caterpillars will become immune to the virus.The sweep of narrative in A. N. Wilson’s biography of C. S. Lewis is impressive and there is much that is acute and well argued. But much in this work is careless and unworthyof its author. Wilson, a novelist and an accomplished biographer, has failed to do what any writer on such a subject as Lewis ought to do, namely work out a coherent view of how the various literary works by the subject are to be described and commented on. Decisions have to be made on what to look at in detail and what to pass by with just a mention. Wilson has not thought this problem out. For instance, Till We Have Faces, Lewis’treatment of the Eros and Psyche story and one of his best-executed and most moving works, is merely mentioned by Wilson, though it illuminates Lewis’ spiritual development, whereas Lewis’ minor work Pilgrim’s Regress is looked at in considerable detail.24. The author of the passage implies that Wilson’s examination of Pilgrim’s Regress(A) is not as coherent as his treatment of Till We Have Faces(B) would have been more appropriate in a separate treatise because of the scope ofPilgrim’s Regress(C) demonstrates how Wilson’s narrow focus ignores the general themes of Lewis’works(D) was more extensive than warranted because of the relative unimportance ofPilgrim’s Regress(E) was disproportionately long relative to the amount of effort Lewis devoted towriting Pilgrim’s Regress25. The author of the passage would be most likely to agree with which of the followingstatements regarding Till We Have Faces?(A) It is an improvement over the Eros and Psyche story on which it is based.(B) It illustrated Lewis’ attempt to involve his readers emotionally in the story ofEros and Psyche.(C) It was more highly regarded by Wilson than by Lewis himself.(D) It is one of the outstanding literary achievements of Lewis’ career.(E) It is probably one of the most popular of Lewis’ works.26. Which of the following best describes the organization of the passage?(A) An evaluation is made, and aspects of the evaluation are expanded on withsupporting evidence.(B) A theory is proposed, and supporting examples are provided.(C) A position is examined, analyzed, and rejected.(D) A contradiction is described, then the points of contention are evaluated andreconciled.(E) Opposing views are presented and evaluated, then modifications are advocated.27. Which of the following best describes the content of the passage?(A) A critique of A. N. Wilson as a biographer(B) An evaluation of the significance of several works by C. S. Lewis(C) An appraisal of a biography by A. N. Wilson(D) A ranking of the elements necessary for a well-structured biography(E) A proposal for evaluating the literary merits of the works of C. S. Lewis1995年04月SECTION AInfluenced by the view of some twentieth-century feminists that women’s position within the family is one of the central factors determining women’s social position, some historians have underestimated the significance of the woman suffrage movement. These historians contend that nineteenth-century suffragist was less radical and, hence, less important than, for example, the moral reform movement or domestic feminism—two nineteenth-century movements in which women struggled for more power and autonomy within the family. True, by emphasizing these struggles, such historians have broadened the conventional view of nineteenth-century feminism, but they do a historical disservice to suffragism. Nineteenth-century feminists and anti-feminist alike perceived the suffragists’demand for enfranchisement as the most radical element in women’s protest, in part because suffragists were demanding power that was not based on the institution of the family, women’s traditional sphere. When evaluating nineteenth-century feminism as a social force, contemporary historians should consider the perceptions of actual participants in the historical events.17. The author asserts that the historians discussed in the passage have(A) influenced feminist theorists who concentrate on the family(B) honored the perceptions of the women who participated in the women suffragemovement(C) treated feminism as a social force rather than as an intellectual tradition(D) paid little attention to feminist movements(E) expanded the conventional view of nineteenth-century feminism18. The author of the passage asserts that some twentieth-century feminists haveinfluenced some historians view of the(A) significance of the woman suffrage movement(B) importance to society of the family as an institution(C) degree to which feminism changed nineteenth-century society(D) philosophical traditions on which contemporary feminism is based(E) public response to domestic feminism in the nineteenth century19. The author of the passage suggests that which of the following was true ofnineteenth-century feminists?(A) Those who participated in the moral reform movement were motivatedprimarily by a desire to reconcile their private lives with their public positions.(B) Those who advocated domestic feminism, although less visible than thesuffragists, were in some ways the more radical of the two groups.(C) Those who participated in the woman suffrage movement sought social roles forwomen that were not defined by women’s familial roles.(D) Those who advocated domestic feminism regarded the gaining of moreautonomy within the family as a step toward more participation in public life.(E) Those who participated in the nineteenth-century moral reform movement stoodmidway between the positions of domestic feminism and suffragism.20. The author implies that which of the following is true of the historians discussed inthe passage?(A) They argue that nineteenth-century feminism was not as significant a social forceas twentieth-century feminism has been.(B) They rely too greatly on the perceptions of the actual participants in the eventsthey study.(C) Their assessment of the relative success of nineteenth-century domestic feminismdoes not adequately take into account the effects of antifeminist rhetoric.(D) Their assessment of the significance of nineteenth-century suffragism differsconsiderably from that of nineteenth-century feminists.(E) They devote too much attention to nineteenth-century suffragism at the expenseof more radical movements that emerged shortly after the turn of the century.Many objects in daily use have clearly been influenced by science, but their form and function, their dimensions and appearance, were determined by technologists, artisans, designers, inventors, and engineers—using non-scientific modes of thought. Many features and qualities of the objects that a technologist thinks about cannot be reduced to unambiguous verbal descriptions; they are dealt with in the mind by a visual, nonverbal process. In the development of Western technology, it has been non-verbal thinking, by and large, that has fixed the outlines and filled in the details of our material surroundings. Pyramids, cathedrals, and rockets exist not because of geometry or thermodynamics, but because they were first a picture in the minds of those who built them.The creative shaping process of a technologist’s mind can be seen in nearly every artifact that exists. For example, in designing a diesel engine, a technologist might impress individual ways of nonverbal thinking on the machine by continually using an intuitive sense of rightness and fitness. What would be the shape of the combustion chamber? Where should the valves be placed? Should it have a long or short piston? Such questions have a range of answers that are supplied by experience, by physical requirements, by limitations of available space, and not least by a sense of form. Some decisions, such as wall thickness and pin diameter, may depend on scientific calculations, but the nonscientific component of design remains primary.Design courses, then, should be an essential element in engineering curricula. Nonverbal thinking, a central mechanism in engineering design, involves perceptions, the stock-in-trade of the artist, not the scientist. Because perceptive processes are not assumed to entail “hard thinking,” nonverbal thought is sometimes seen as a primitive stage in the development of cognitive processes and inferior to verbal or mathematical thought. But it is paradoxical that when the staff of the Historic American EngineeringRecord wished to have drawings made of machines and isometric views of industrial processes for its historical record of American engineering, the only college students with the requisite abilities were not engineering students, but rather students attending architectural schools.If courses in design, which in a strongly analytical engineering curriculum provide the background required for practical problem-solving, are not provided, we can expect to encounter silly but costly errors occurring in advanced engineering systems. For example, early models of high-speed railroad cars loaded with sophisticated controls were unable to operate in a snowstorm because a fan sucked snow into the electrical system. Absurd random failures that plague automatic control systems are not merely trivial aberrations; they are a reflection of the chaos that results when design is assumed to be primarily a problem in mathematics.21. In the passage, the author is primarily concerned with(A) identifying the kinds of thinking that are used by technologists(B) stressing the importance of nonverbal thinking in engineering design(C) proposing a new role for nonscientific thinking in the development of technology(D) contrasting the goals of engineers with those of technologists(E) criticizing engineering schools for emphasizing science in engineering curricula22. It can be inferred that the author thinks engineering curricula are(A) strengthened when they include courses in design(B) weakened by the substitution of physical science courses for courses designed todevelop mathematical skills(C) strong because nonverbal thinking is still emphasized by most of the courses(D) strong despite the errors that graduates of such curricula have made in thedevelopment of automatic control systems(E) strong despite the absence of nonscientific modes of thinking23. Which of the following statements best illustrates the main point of lines 1-28 of thepassage?(A) When a machine like a rotary engine malfunctions, it is the technologist who isbest equipped to repair it.(B) Each component of an automobile—for example, the engine or the fuel tank—hasa shape that has been scientifically determined to be best suited to thatcomponent’s function.(C) A telephone is a complex instrument designed by technologists using onlynonverbal thought.(D) The designer of a new refrigerator should consider the designs of otherrefrigerators before deciding on its final form.(E) The distinctive features of a suspension bridge reflect its designer’sconceptualization as well as the physical requirements of its site.。

GRE阅读真题之OG汇总

GRE阅读真题之OG汇总

GRE阅读真题之OG汇总想要在GRE阅读中拿高分,了解阅读真题是必需的,下面我就和大家共享,来欣赏一下吧。

GRE阅读真题之OGOG-1Passage 5Since the Hawaiian Islands have never been connected to other land masses, the great variety of plants in Hawaii must be a result of the long-distance dispersal of seeds, a process that requires both a method of transport and an equivalence between the ecology of the source area and that of the recipient area.There is some dispute about the method of transport involved. Some biologists argue that ocean and air currents are responsible for the transport of plant seeds to Hawaii. Yet the results of flotation experiments and the low temperatures of air currents cast doubt on these hypotheses. More probable is bird transport, either externally, by accidental attachment of the seeds to feathers, or internally, by the swallowing of fruit and subsequent excretion of the seeds. While it is likely that fewer varieties of plant seeds have reached Hawaii externally than internally, more varieties are known to be adapted to external than to internal transport.1. The author of the passage is primarily concerned withA. discussing different approaches biologists have taken to testing theories about the distribution of plants in HawaiiB. discussing different theories about the transport of plant seeds to HawaiiC. discussing the extent to which air currents are responsible for the dispersal of plant seeds to HawaiiD. resolving a dispute about the adaptability of plant seeds to bird transportE. resolving a dispute about the ability of birds to carry plant seeds long distances2. The author mentions the results of flotation experiments on plant seeds (lines 7–8) most probably in order toA. support the claim that the distribution of plants in Hawaii is the result of the long-distance dispersal of seedsB. lend credibility to the thesis that air currents provide a method of transport for plant seeds to HawaiiC. suggest that the long-distance dispersal of seeds is a process that requires long periods of timeD. challenge the claim that ocean currents are responsible for the transport of plant seeds to HawaiiE. refute the claim that Hawaiian flora evolved independently from flora in other parts of the worldGRE阅读真题之OGOG-1Passage 6I enjoyed A Dream of Light Shadow: Portraits of Latin American Women Writers for the same reasons that, as a child, I avidly consumed women’s biographies:the fascination with how the biographical details of another female’s life are represented and interpreted.A Dream offers a rich read, varied in both the lives and texts of the women portrayed, and the perspectives and styles of the sixteen essayists. Yet, as an adult, I have come to demand of any really “great” book a self-consciousness about the tenuous nature of representations of reality, a critical contextualization of florid detail, and a self-awareness of the role of ideology in our lives. In these critical senses, A Dream is inadequate.For the following question, consider each of the choices separately and select all that apply.1. The author of the passage suggests that A Dream falls short in which of the following respects?A. It does not appear to recognize that representations of reality can be unreliable.B. It seems to focus on stylistic variety at the expense of accuracy of detail.C. It offers a wealth of detail without sufficient critical examination of that detail.2. Which of the following best describes the function of the second sentence (“A Dream . . . essayists”) in the context of the passage as a whole?A. To give examples of how A Dream presents fascinating portraits that display awareness of the tenuous nature of representations of realityB. To elaborate on how A Dream fulfills the author’s childhood criteria for a pleasurable bookC. To suggest that the author enjoyed A Dream for reasons more sophisticated than the reasons she enjoyed certain books as a childD. To illustrate ways in which the author finds A Dream to be inadequate in certain critical sensesE. To imply that A Dream is too varied in focus to provide a proper contextualization of the biographical details it offersGRE阅读真题之OGGRE阅读真题之OG Passage 72021-09-21 17:25来源:新东方网整理:共享到新东方网GRE频道在此和大家共享GRE官方指南中的阅读真题,希望对大家预备GRE阅读有所关怀。

新GRE逻辑阅读练习题汇总

新GRE逻辑阅读练习题汇总

新GRE逻辑阅读练习题汇总新GRE逻辑阅读练习题Hydrogeology is a science dealing with the properties, distribution, and circulation of water on the surface of the land, in the soil and underlying rocks, and Line in the atmosphere. The term “geohydrology" is 5 sometimes erroneously used as a synonym for "hydrogeology". Geohydrology is concerned with underground water. There are many formations that contain water but are not part of the hydrologic cycle because of geologic changes that have isolated them 10 underground. These systems are properly termed geohydrologic but not hydrogeologic. Only when a system possesses natural or artificial boundaries that associate the water within it with the hydrologic cycle may the entire system properly be termed hydrogeologic. (107 words)1. It can be inferred that which of the following is most likely to be the subject of study by a geohydrologist?(A) Soft, porous rock being worn away by a waterfall(B) Water depositing minerals on the banks of a gorge through which the water runs(C) The trapping of water in a sealed underground rockcavern through the actionof an earthquake(D) Water becoming unfit to drink through the release of pollutants into it from a manufacturing plant(E) The changing course of a river channel as the action of the water wears away the rocks past which the river flows2. The author refers to "many formations" primarily in order to(A) clarify a distinction(B) introduce a subject(C) draw an analogy(D) emphasize a similarity(E) resolve a conflict答案:C/A新GRE逻辑阅读练习题Initially the Vinaver theory that Malory‘s eight romances, once thought to be fundamentally unified. were in fact eight independent works produced both a Line sense of relief and an unpleasant shock. Vinaver‘s 5 th eory comfortably explained away the apparent contradictions of chronology and made each romance independently satisfying. It was, however, disagreeableto find that what had been thought of as one book was now eight books. Part of this response was the natural 10 reaction to the disturbance of set ideas. Nevertheless, even now, after lengthy consideration of the theory‘s refined but legitimate observations, one cannot avoid the conclusion that the eight romances are only one work. It is not quite a matter of disagreeing with the 15 theory of independence, but of rejecting its implications: that the romances may be taken in any or no particular order, that they have no cumulative effect, and that they are as separate as the works of a modern novelist. (154 words)For the following question, consider each of the choices separately and select all that apply5. It can be inferred from the passage that the author believes which of the following about Malory‘sworks?□A There are meaningful links be tween and among the romances.□B The subtleties of the romances are obscured when they are taken as one work.□C Any contradictions in chronology among the romances are less important than their over-all unity.6. The author of the passage concedes which of the following about the Vinaver theory?(A) It gives a clearer understanding of the unity of Malory‘s romances.(B) It demonstrates the irrationality of considering Malory‘s romances to be unified.(C) It establishes acceptab le links between Malory‘s romances and modern novels.(D) It unifies earlier and later theories concerning the chronology of Malory‘s romances.(E) It makes valid and subtle comments about Malory‘s romances.7. Select the sentence in the passage that suggests that, in evaluating the Vinaver theory, some critics were initially biased by previous interpretations of Malory‘s work.答案及解析:第一题选AC做这个题首先要对文章进行剖析。

GRE阅读36套(7)节选

GRE阅读36套(7)节选

GRE阅读36套(7)节选GRE阅读36套(7)节选1.1HG是一门研究水在陆地表面、土壤以及岩石和大气中性质、分配和流通的科学。

GH有时候会被错误地被当作HG的同义词。

GH主要关注的地下水。

有很多形态的水并不属于水圈因为地质变化将这些水隔离在地下。

这一系统更适合称为GH而不是HG。

只用当一个系统具备了自然或是人工界限并将其中的水和水圈中的水联系起来,这样整个系统才更适合称为HG。

1.216世纪50年代,当沙龙已经在法国很稳固地建立起来的时候,一些英国妇女,她们自成为蓝袜以法国S(法国沙龙女主人)为样板建立了自己的沙龙。

大多数蓝袜女们并不希望自己的沙龙是法国S的翻版,他们只是希望使那些已经通过的原则适应她们的目的,即通过道德和智能的训练提高妇女地位。

社会方向和社会背景的不同或许是法国和英国沙龙性质上不同的原因。

法国沙龙代表了贵族的态度,即提高优雅愉悦,强调艺术成就。

而英国蓝袜女们,出身更普通的背景,强调学习和工作而不是愉悦。

习惯了宫廷圈子刻板的生活,S的沙龙非常正式,而英国妇女的沙龙,尽管也有些拘谨,但是在方式上更随意些。

1.3热泵的使用被大打折扣由于人们会广告宣传的怀疑。

这些广告声称热泵可以提供,对于消耗的每个单位的电能来说,热泵可产生两个单位的热能,因此显然与能量守恒定理相悖。

1.4FD所有现存的作品混合了肖像画和风俗画。

她的主人公都是一些她熟知的人,她让他们摆出姿势;她捕捉到他们日常生活中的内心自我意识和自然状态,这就是风俗画作的典型表现手法。

但是风俗画,至少描绘低阶层人的那些风格化,在18世纪的法国从来没有流行过。

LN 兄弟和G,他们都选择了这样的题材,但是都被忽略掉了。

他们现在获得了较高的评价是因为更民主的政治氛围和不用的审美价值有别于当时,即我们不再需要艺术家提供人类完美的形象作为我们道德楷模,我们现在认为这样的完美是对真实的作假。

FD从不表达粉饰现实的主题,并小心翼翼地避免对其人物对象作评判。

GRE考试《阅读理解》练习题及答案(2)

GRE考试《阅读理解》练习题及答案(2)

GRE考试《阅读理解》练习题及答案(2)GRE阅读题目解析:弗兰德斯派大师Stylistic evidence and laboratory evidence strongly support the claim that the magnificent painting Garden of Eden is a work of the Flemish master van Eyc k. Nevertheless, the painting must have been the work of someone else, as anyon e with a little historical and zoological knowledge can tell merely by looking at the painting. The animals in the painting are all vivid representations of a ctual animals, including armadillos. Yet armadillos are native only to America s, and van Eyck died decades before Europeans reached the Americas.2. In the argument given, the two highlighted portions play which of the fo llowing roles?A. The first is a position that the argument seeks to reject, the second is evidence that the argument uses against that position.B. The first and the second are each pieces of evidence that have been used to support the position that the argument opposes.C. The first presents the main conclusion of the argument; the second provi des evidence in support of that conclusion.D. The first is a judgment that serves as the basis for the main conclusion of the argument; the second states that main conclusion.E. The first is an intermediate conclusion drawn in order to support a furt her conclusion stated in the argument; the second provides evidence in support of that intermediate conclusion.1Stylistic evidence and laboratory evidence strongly support the claim that the magnificent painting Garden of Eden is a work of the Flemish master van Eyc k.风格上的和来自实验室的证据,都有力地支持这样的观点,即大画 G o E 出自弗兰德斯派大师 v E 之手。

gre阅读真题的翻译和解析

gre阅读真题的翻译和解析

gre阅读真题的翻译和解析1.一种曾在19世纪造成白马铃薯饥荒的真菌的新的更有侵略性的类型在最近出现。

然而由于这种真菌可以被加大用量的杀菌剂杀死,因而这种真菌不会带来对那些依赖于马铃薯为食物的国家造成大面积的食物短缺。

问题:下面哪一个会给文中的结论带来疑问?A.尽管马铃薯是世界上很多地方的主要农作物,在大多数国家人们首要的还是以小麦或者大米为食。

B.在新型真菌扩散的大多数国家农民不能增加他们在杀菌剂上的花费。

C.这种新的真菌是在被感染的马铃薯从一个马铃薯主要出口国出口过来之后开始传播的D.在大多数国家马铃薯农除了使用杀死这种新型真菌的杀菌剂外还使用了几种杀虫剂E.大多数政府留出了可以减轻类似于食物短缺和洪灾之类的大规模灾害的影响的资金。

答案选B,毋庸置疑。

2.作为机关,面临着预算困难,减少科学研究的支持,更大数量的类似研究是由私人基金会支持的。

这种转变意味着会带来有争议的科学研究项目会在所有的支持的研究项目中占更少的部分,因为私人基金会考虑到自己的公共形象而趋向于避免争议。

问题:下面哪一个是上面的假设?A.只有那些不会有可能带来争议顾虑而进行的研究会产生科学上有效的结果。

B.支持科学研究项目的私人基金会意识到项目可能的争议是不可总避免的。

C.有私人基金会支持的项目科学家不同意支持机构在他们做研究时影响他们的做法。

D.许多的政府机构比私人基金会更在意自己的公共形象。

E.政府机构比私人机构更愿意支持会产生争议的科研项目。

答案选E,A无关,B相反,C也相反,D也是。

3.刑事审判陪审团不会依据任何目击者没有证实的证词做出裁决,这样很对,因为对任何人做出的未经证实的说法保持高度的怀疑是谨慎的。

但是为做到始终如一,陪审团应该终止一种太寻常的行为:在未经证实的供认基础上对被告指控有罪。

问题:下面哪一个加强了上面的结论?A.陪审团经常在那些被告收回在审讯之前做的所有供认的案子中宣判无罪。

B.陪审团选举的过程中会去掉那些在审讯之前就对被告的犯罪有坚定的观点的人。

新gre阅读逻辑10套及答案解析

新gre阅读逻辑10套及答案解析

新gre阅读逻辑10套及答案解析可能有的小伙伴们在备考gre阅读的时候一直认为阅读就得需要大题量的练习来进行备考,但是对gre阅读考试的相关内容缺乏了解,今天小编为大家带来的资料就是新gre阅读逻辑10套及答案解析。

新gre阅读逻辑10套1. Armtech, a temporary-employment agency, previously gave its employees2.5 paid vacation days after each 700 hours worked. Armtech's new policy is to give its employees 5.0 paid vacation days after each 1,200hours worked. Therefore, this new policy is more generous to Armtech employees in giving them more vacation days per hour worked than the old policy did. Which of the following is an assumption on whichthe argument depends?(A) Most current Armtech employees approve of the company's new vacation policy.(B) A few Armtech employees leave the company before having worked 700 hours.(C) Most Armtech employees were not aware that the company planned to change its vacation policy until after it had already done so.(D) A significant portion of Armtech employees stay with the company long enough to work for 1,200 hours.(E) Armtech's new vacation policy closely matches the vacation policies of competing temporary employment agencies.2 The global population of frogs has declined in recent years while the amount of ultraviolet radiation reaching the Earth has increased. Since the genetic material infrog eggs is harmed when exposed to ultraviolet radi- ation,and since the eggs themselves are not protected by shells or leathery coverings but are gelatinous, the frog population decline is probably due, at least in part, to the ultraviolet radiation increase. Which of the following, if true, provides the strongest support for the argument?(A) Even in those regions where there has been no significant increase in ultraviolet radiation, only a small proportion of the frog eggs that are laid ever hatch.(B) In areas where there has been the least decline in frog populations, populations of species of insects that frogs eat have decreased.(C) The eggs of frog species whose populations are declining tend to have higher concentrations of damaging pesticides than do the eggs of frog species whose populations have not declined.(D) In many places where turtles, which lay eggs with tough, leathery coverings, share habitats with frogs, turtle populations are also in decline.(E) Populations of frog species that hide their eggs beneath rocks or under sand have declined considerably less than have populations of frog species that do not cover their eggs.3 To improve productivity, manufacturing companies have recently begun restructuring work to produce more goods with fewer assembly-line workers, and the companies have laid off many workers as aconsequence. The workers laid off have been those with the least seniority(time on the job), generallythe younger workers.The statements above, if true, most strongly support which of the following as a conclusion?(A) The products manufactured by the companies are not undergoing design changes while the manufacturing jobs arebeing restructured.(B) When assembly-line workers have made sug- gestions for improvements in manufacturing processes, some suggestions have been implemented, but many have not.(C) Assembly-line workers now need increased reading and mathematical skills to do their jobs.(D) Some of the innovations in assembly-line processes and procedures that were made to increase productivity have instead proved to be counterproductive.(E) The manufacturing companies are increasing the average age of their assembly-line workforce while still seeking to increase production.4.During the nineteenth century, Britain's urban popu- lation increased as its rural population diminished. A historian theorizes that, rather than industrialization'sbeing the cause, this change resulted from a series of migrations to urban areas, each occasioned by a depression in the agrarian economy. To test this hypoth-esis, the historian will compare economic data with population census data. The historian's hypothesis would be most strongly supported if which of the following were found to betrue?(A) The periods of greatest growth in the industrial economy were associated with a relatively rapid decline in the rural population.(B) The periods of greatest weakness in the agrarian economy were associated with relatively slow growth in the population as a whole.(C) Periods when the agrarian economy was compar- ativelystrong and the industrial economy com- paratively weak were associated with a particu- larly rapid decline in the rural population.(D) Periods when the agrarian and industrial econo- mies were both strong were associated with particularly rapid growth in the urban popula- tion.(E) The periods of greatest strength in the agrarian economy were associated with relatively slow growth in the urban population.5 Politician:Each year, small businesses create more jobs than do large established businesses. Therefore, in order to reduce unemployment in the long term, we should provide incentives for starting small businesses rather than for expanding established large businesses. Which of the following, if true, casts the most doubt on the politician's argument?(A) In general, people employed by small businesses report higher job satisfaction than do people employed by large businesses.(B) Among the currently unemployed are many people with sufficient job skills to perform the jobs that small businesses would create.(C) Providing an effective incentive for starting a business generally costs significantly less than providing an effective incentive for expanding a large business.(D) A high proportion of small businesses fail within three years of starting because of their owners' inexperience.(E) The average large business contributes more money to politicians’ campaign funds than the average small business does.新GRE阅读逻辑10套答案1.内容:尽管在该地区的高中入学率已经连续几年下降,但小学的入学率已经有相当大地增长,因此,地区教育委员会建议建立一所新的小学。

短文阅读练习题及参考答案GRE考试

短文阅读练习题及参考答案GRE考试

短文阅读练习题及参考答案GRE考试短文阅读练习题及参考答案GRE考试「篇一」《穷人》片段渔夫的妻子桑娜坐在火炉旁边补一张破帆。

屋外寒风(呼啸怒吼),(波涛汹涌汹涌澎湃)的海浪(拍击打击)着海岸,(溅掀)起一阵阵浪花。

海上正起着(暴风风暴),外面有黑又冷,这间渔家的小屋里却(暖和温暖)而(舒服舒适),地扫得干干净净,炉子里的火好没有熄,食具在搁板上(闪闪发光闪闪发亮)。

挂着白色帐子的床上,五个孩子正在海风呼啸中(平静安静)地睡着。

丈夫清早驾着小船出海,这时候还没有回来。

桑娜听着波涛的(汹涌轰鸣)和狂风的(怒吼呼啸),感到(胆战心惊心惊肉跳)。

1.划掉括号中不恰当的词语。

2.从这段话可以知道,桑娜一家生活是______的,从_____可以看出来;桑娜是一个______的人,从_____和______可以看出来;桑娜这时的心情是_______,因为___________________。

参考答案1.怒吼波涛汹涌打击掀暴风暖和舒服闪闪发光平静汹涌呼啸胆战心惊2.贫穷的从第一句可以看出勤劳地扫得干干净净,食具在搁板上闪闪发亮非常焦急的丈夫清早驾着小船出海,这时候还没有回来短文阅读练习题及参考答案GRE考试「篇二」一篇小升初语文阅读短文练习题及答案小升初语文阅读短文练习题(含答案)人生的标点我的一生是一句话,母亲最希望我把它推敲得美丽些、生动些。

我用我的脚印落成文字,将它书写在时间的稿笺上。

后人为我堆起的那座坟墓,想来就是句末的标点了。

那么,我将表达出怎样的意思呢?我将脚印首先落成“善”字——我觉得它是最不可缺少的。

母亲发现后,点了点头,却又说:“这还不够,后人只能在句末给你点成逗号。

”但我渴慕一个表示完整的句子。

于是我又将脚印落成“信”字——我觉得它是最靠得住的。

母亲发现后,点了点头,却又说:“这还不够,后人只能在句末给你点成逗号。

”于是我将脚印落成“顽强”,落成“进取”,落成“创造”——我觉得它们都是闪闪发光的。

gre阅读理解真题206篇

gre阅读理解真题206篇

gre阅读理解真题206篇During adolescence,the development of politicalideology beco mes apparent in the individual;ideology here is defined as the presence ofroughly consistent attitudes,more or lessorganize d in reference to a more encompassing,though perhaps tacit,set of general principles.Assuch,political ideology is dim or abs ent at thebeginning of adolescence.Its acquisition by theadol escent,in even the most modest sense,requires the acquisition of relativelysophisticated cognitive skills:the ability to ma nage abstractness to synthesize andgeneralize,to imagine the future.These are accompanied by a steady advance in the ability to understand principles.The child's rapid acquisition of political knowledge also p romotes the growth of politicalideology during adolescence.By knowledge I mean more than the dreary"facts,"such as thecompos ition of county government that the child is exposed to in the co nventional ninth-grade civics course.Nor do I mean only information on current po litical realities.These arefacets of knowledge,but they are le ss critical than the adolescents absorption,oftenunwitting,of a feeling for those many unspoken assumptions about the politic al system thatcomprise the common ground of understandingfor example,what the state can"appropriately"demand of its citizens ,and vice versa,or the"proper"relationship ofgovernment to s ubsidiary social institutions,such as the schools and churches .Thus,politicalknowledge is the awareness of social assumption s and relationships as well as of objectivefacts.Much of the nai vete that characterizes the younger adolescent's grasp of polit ics stemsnot from an ignorance of"fact"but from an incomplete c omprehension of the commonconventions of the system,of what is and is not customarily done,and of how and why it is or isnot don e.Yet I do not want to overemphasize the significance of incre ased political knowledge informing adolescent ideology.Over th e years I have become progressively disenchanted aboutthe centr ality of such knowledge and have come to believe that much curren t work in politicalsocialization,by relying too heavily on its apparent acquisition,has been misled about thetempo of politic al understanding in adolescence Just as young children can count numbers inseries without grasping the principle of ordination, young adolescents may have in theirheads many random bits of pol itical information without a secure understanding of thoseconce pts that would give order and meaning to the information.Like magpies,children's minds pick up bits and pieces of data.if you encourage them,theywill drop these at your feet—Republicans and Democrats,the tripartite division of the feder alsystem,perhaps even the capital of Massachusetts.But until t he adolescent has grasped theintegumental function that concept s and principles provide,the data remain fragmented,random,di sordered.The author's primary purpose in the passage is toA.clarify the kinds of understanding an adolescent must have in order to develop a politicalideologyB.dispute the theory that a political ideology can be acquired d uring adolescenceC.explain why adolescents are generally uninterested in politic al argumentsD.suggest various means of encouraging adolescents to develop p ersonal political ideologiesE.explain why an adolescent's political ideology usually appear s more sophisticated than itactually isAccording to the author,which of the following contributes to the development ofpolitical ideology during adolescence? A.Conscious recognition by the adolescent of his or her own naiv eteB.Thorough comprehension of the concept of ordinationC.Evaluation by the adolescent of the general principles encomp assing his or her specificpolitical ideasD.Intuitive understanding of relationships among various compo nents of societyE.Rejection of abstract reasoning in favor of involvement with p ragmatic situationsThe author uses the term"common ground of understanding" (lines36-37)to refertoA.familiar legislation regarding political activityB.the experiences that all adolescents shareC.a society's general sense of its own political activityD.a society's willingness to resolve political tensionsE.the assumption that the state controls social institutionsThe passage suggests that,during early adolescence,a chil d would find which of thefollowing most difficult to understand?A.A book chronicling the ways in which the presidential inaugura tion ceremony has changedover the yearsB.An essay in which an incident in British history is used to exp lain the system of monarchicsuccessionC.A summary of the respective responsibilities of the legislati ve,executive,and judicialbranches of governmentD.A debate in which the participants argue,respectively,that the federal government should orshould not support private schoo lsE.An article detailing the specific religious groups that found ed American colonies and theguiding principles of each one It can be inferred from the passage that the author would be most likely to agree withwhich of the following statements about schools?A.They should present political information according to carefu lly planned,schematicarrangements.B.They themselves constitute part of a general sociopolitical s ystem that adolescents arelearning to understand.C.If they were to introduce political subject matter in the prim ary grades,students wouldunderstand current political realiti es as an earlier age.D.They are ineffectual to the degree that they disregard adolesc ents'political naivete.E.Because they are subsidiary to government,their contribution to the politicalunderstanding of adolescents must be limited.Which of the following best summarizes the author's evaluat ion of the accumulationof political knowledge by adolescents?A.It is unquestionably necessary,but its significance can easi ly be overestimated.B.It is important,but not as important as is the ability to appe ar knowledgeable.C.It delays the necessity of considering underlying principles.D.It is primarily relevant to an understanding of limited,local concerns,such as countrypolitics.E.It is primarily dependent on information gleaned from high sch ool courses such as civics.Which of the following statements best describes the organi zation of the author'sdiscussion of the role of political knowle dge in the formation of political ideologyduring adolescence?A.He acknowledges its importance,but then modifies his initial assertion of that importance.B.He consistently resists the idea that it is important,using a series of examples to supporthis stand.C.He wavers in evaluating it and finally uses analogies to expla in why he is indecisive.D.He beings by questioning conventional ideas about its importa nce,but finally concedesthat they are correct.E.He carefully refrains from making an initial judgment about it ,but later confirms its criticalrole.。

GRE阅读真题之OG精选

GRE阅读真题之OG精选

GRE阅读真题之OG精选我整理了GRE阅读真题,盼望对大家有关心,下面我就和大家共享,来观赏一下吧。

GRE阅读真题之OGOG-1Passage 20Sparva, unlike Treland’s other provinces, requires automobile insurers to pay for any medical treatment sought by someone who has been involved in an accident; in the other provinces, insurers pay for nonemergency treatment only if they preapprove the treatment. Clearly, Sparva’s less restrictive policy must be the explanation for the fact that altogether insurers there pay for far more treatments after accidents than insurers in other provinces, even though Sparva does not have the largest population.1. Which of the following, if true, most strengthens the argument?A. Car insurance costs more in Sparva than in any other province.B. The cost of medical care in Sparva is higher than the national average.C. Different insurance companies have different standards for determining what constitutes emergency treatment.D. Fewer insurance companies operate in Sparva than in any otherprovince.E. There are fewer traffic accidents annually in Sparva than in any of the provinces of comparable or greater population.GRE阅读真题之OGOG-1Passage 21Elements of the Philosophy of Newton, published by Voltaire in 1738, was an early attempt to popularize the scientific ideas of Isaac Newton. In the book’s frontispiece, Voltaire is seen writing at his desk, and over him a shaft of light from heaven, the light of truth, passes through Newton to Voltaire’s collaborator Madame du Châtelet; she reflects that light onto the inspired Voltaire. Voltaire’s book commanded a wide audience, according to Feingold, because “he was neither a mathematician nor a physicist, but a literary giant aloof from the academic disputes over Newtonian ideas.” In other words, Voltaire’s amateurism in science “was a source of his contemporary appeal, demonstrating for the first time the accessibility of Newton’s ideas to nonspecialists.”For the following question, consider each of the choices separately and select all that apply.1. Which of the following statements about Voltaire’s Elements of the Philosophy of Newton can be inferred from the passage?A. Voltaire’s literary stature helped secure a large audience for this attempt to popularize Newton’s ideas.B. Voltaire’s status as a nonscientist was an advantage in this effort to bring Newtonian science to the attention of the general public.C. The frontispiece of the book implies that Voltaire’s understanding of Newton’s ideas was not achieved without assistance.2. Select the sentence that describes an allegory for Voltaire’s acquisition of knowledge concerning Newton’s ideas.GRE阅读真题之OGOG-1Passage 22It would be expected that a novel by a figure as prominent as W. E.B. DuBois would attract the attention of literary critics. Additionally, when the novel subtly engages the issue of race, as DuBois’ The Quest of the Silver Fleece (1911) does, it would be a surprise not to encounter an abundance of scholarly work about that text. But though valuable scholarship has examined DuBois’ political and historical thought, his novels have received scant attention. Perhaps DuBois the novelist must wait his turn behind DuBois the philosopher, historian, and editor. But what if the truth lies elsewhere: what if his novels do not speak to current concerns?1. Which of the following can be inferred from the passageregarding DuBois’ The Quest of the Silver Fleece?A. The lack of attention devoted to The Quest of the Silver Fleece can be attributed to the fact that it was DuBois’ first novel.B. Among DuBois’ novels, The Quest of the Silver Fleece is unusual in that it has received scant attention from scholars.C. The Quest of the Silver Fleece has at least one feature that typically would attract the attention of literary scholars.D. The Quest of the Silver Fleece, given its subtle exploration of race, is probably the best novel written by DuBois.E. Much of the scholarly work that has focused on The Quest of the Silver Fleece has been surprisingly critical of it.2. In the fourth sentence (“Perhaps DuBois . . . editor.”), the author of the passage is most likely suggesting thatA. scholars will find that DuBois’ novels are more relevant to current concerns than is his work as philosopher, historian, and editorB. more scholarly attention will be paid to The Quest of the Silver Fleece than to DuBois’ other novelsC. DuBois’ novels will come to overshadow his work as philosopher, historian, and editorD. DuBois’ novels may eventually attract greater scholarly interest than they have to dateE. it will be shown that DuBois’ work as philosopher, historian, andeditor had an important influence on his work as novelist3. Which of the following best describes the central issue with which the passage is concerned?A. The perfunctoriness of much of the critical work devoted to DuBois’ novelsB. The nature of DuBois’ engagement with the issue of race in The Quest of the Silver FleeceC. Whether DuBois’ novels are of high quality and relevant to current concernsD. The relationship between DuBois the novelist and DuBois the philosopher, historian, and editorE. The degree of consideration that has been given to DuBois’ novels, including The Quest of the Silver FleeceGRE阅读真题之OGOG-1Passage 23Saturn’s giant moon Titan is the only planetary satellite with a significant atmosphere and the only body in the solar system other than Earth that has a thick atmosphere dominated by molecular nitrogen. For a long time, the big question about Titan’s atmosphere was how it could be so thick, given that Jupiter’s moons Ganymede and Callisto, which are the same size as Titan, have none. The conditions for acquiring andretaining a thick nitrogen atmosphere are now readily understood. The low temperature of the protosaturnian nebula enabled Titan to acquire the moderately volatile compounds methane and ammonia (later converted to nitrogen) in addition to water. The higher temperatures of Jupiter’s moons, which were closer to the Sun, prevented them from acquiring such an atmosphere.1. According to the passage, Titan differs atmospherically from Ganymede and Callisto because of a difference inA. rate of heat lossB. proximity to the SunC. availability of methane and ammoniaD. distance from its planetE. sizeGRE阅读真题之OGOG-1Passage 24Observations of the Arctic reveal that the Arctic Ocean is covered by less ice each summer than the previous summer. If this warming trend continues, within 50 years the Arctic Ocean will be ice free during the summer months. This occurrence would in itself have little or no effect on global sea levels, since the melting of ice floating in water does not affect the water level. However, serious consequences to sea levelswould eventually result, because __________.1. Which of the following most logically completes the passage?A. large masses of floating sea ice would continue to form in the wintertimeB. significant changes in Arctic sea temperatures would be accompanied by changes in sea temperatures in more temperate parts of the worldC. such a warm Arctic Ocean would trigger the melting of massive landbased glaciers in the ArcticD. an ice-free Arctic Ocean would support a very different ecosystem than it does presentlyE. in the spring, melting sea ice would cause more icebergs to be created and to drift south into shipping routesGRE阅读真题之OGOG-1Passage 25In a recent study, David Cressy examines two central questions concerning English immigration to New England in the 1630s: what kinds of people immigrated and why? Using contemporary literary evidence, shipping lists, and customs records, Cressy finds that most adult immigrants were skilled in farming or crafts, were literate, and were organized in families. Each of these characteristics sharply distinguishesthe 21,000 people who left for New England in the 1630s from most of the approximately 377,000 English people who had immigrated to America by 1700.With respect to their reasons for immigrating, Cressy does not deny the frequently noted fact that some of the immigrants of the 1630s, most notably the organizers and clergy, advanced religious explanations for departure, but he finds that such explanations usually assumed primacy only in retrospect. When he moves beyond the principal actors, he finds that religious explanations were less frequently offered, and he concludes that most people immigrated because they were recruited by promises of material improvement.For the following question, consider each of the choices separately and select all that apply.1. The passage indicates that Cressy would agree with which of the following statements about the organizers among the English immigrants to New England in the 1630s?A. Some of them offered a religious explanation for their immigration.B. They did not offer any reasons for their immigration until some time after they had immigrated.C. They were more likely than the average immigrant to be motivated by material considerations.2. Select the sentence that provides Cressy’s opinion about what motivated English immigrants to go to New England in the 1630s.“When … movement.”3. In the passage, the author is primarily concerned withAA. summarizing the findings of an investigationB. analyzing a method of argumentC. evaluating a point of viewD. hypothesizing about a set of circumstancesE. establishing categories20E21ABC“In … Voltaire.”22CDE23B24C 25A“When … movement.”A。

新GRE逻辑阅读练习题精选5篇

新GRE逻辑阅读练习题精选5篇

新GRE逻辑阅读练习题精选5篇GRE的阅读考生特别注意规律关系,所以大家要多写一些新GRE 规律阅读练习题,快来一起学习吧,下面我就和大家共享,来观赏一下吧。

新GRE规律阅读练习题For each of Questions 1-3, select one answer choice unless otherwise instructed.Questions 1 to 3 are based on the following reading passage.Paule Marshalls Brown Girl, Brownstones (1959) was a landmark in the depiction of female characters in Black American literature. Marshall avoided the oppressed and tragic heroine in conflict with White society that had been 5 typical of the protest novels of early twentieth century.Like her immediate predecessors, Zora Neale Hurston and Gwendolyn Brooks, she focused her novel on an ordinary Black womans search for identity within the context of a Black community. But Marshall extended the 10 analysis of Black female characters begun by Hurston and Brooks by depicting her heroines development in terms of the relationship between her Barbadian American parents, and by exploring how male and female roles were defined by their immigrant culture, which in turn 15 was influenced by the materialism of White America.By placing characters within a wider cultural context, Marshall attackedracial and sexual stereotypes and paved the way for explorations of race, class, and gender in the novels of the 1970s. For the following question, consider each of the choices separately and select all that apply1. It can be inferred that the author of the passage would describe Brown Girl, Brownstones as being□A highly influenced by novels written in the early wentieth century □B important in the late 1950s but dated today□C an important influence on novels written in the1970s2. According to the passage, Hurston, Brooks, and Marshall are alike in that they○A did not examine the effects of White culture on their characters lives○B were heavily influenced by the protest novels of the early twentieth century○C used Black communities as the settings for their novels.○D wrote primarily about the difficulties their characters encountered in White culture○E wrote exclusively about female characters and the experiences of women3. The author‘s description of the way in which Marshall depicts her heroines development is most probably intended to○A continue the discussion of similarities in the works of Brooks,Hurston, and Marshall○B describe the specific racial and sexual stereo-types that Marshall attacked○C contrast the characters in Marshalls novels with those in later works○D show how Marshall extends the portrayal of character initiated by her predecessors○E compare themes in Marshalls early work with themes in her later novels答案:1. C. 2. C 3 D新GRE规律阅读练习题Calculations of the density of alloys based on Bernal-type models of the alloy‘s metal component agreed fairly well with the experimentally determined values from measurements on alloys consisting of a 5 noble metal together with a metalloid, such as alloys of palladium and silicon, or alloys consisting of iron,phosphorus, and carbon, although small discrepancies remained. One difference between real alloys and the hard spheres used in Bernal models is that the 10 components of an alloy have different sizes, so that models based on two sizes of spheres are more appropriate for a binary alloy, for example. The smaller metalloid atoms of the alloy might fit into holes in the dense, random-packed structure of the larger metal atoms.4. The authors speculation about the appropriateness of models using spheres of two sizes for binary alloys would be strongly supported if models using spheres of two sizes yielded○A values for density identical to values yielded by one-sphere models using the smaller spheres only○B values for density agreeing nearly perfectly with experimentally determined values○C values for density agreeing nearly perfectly with values yielded by models using spheres of three sizes○D significantly different values for density depending on the size ratio between the two kinds of spheres used○E the same values for density as the values for appropriately chosen models that use only medium-sized spheres答案:B新GRE规律阅读练习题Questions 5 and 6 are based on the following reading passage.One of the principal themes of Walzers critique of liberal capitalism is that it is insufficiently egalitarian.Walzers case against the economic inequality generated bycapitalism and in favor of a radical redistribution of 5 wealth is presented in a widely cited essay entitled In Defense of Equality. The most striking feature of Walzers critique is that, far from rejecting the principle of reward according to merit, Walzer insists on itsvalidity.People who excel should receive the superior 10 benefits appropriate to their excellence. But people exhibit a great variety of qualities—intelligence, physical strength, agility and grace, artistic creativity,mechanical skill, leadership, endurance, memory, psychological insight, the capacity for hard 15 work—even moral strength, sensitivity, the ability to express compassion.Each deserves its proper recompense, and hence a proper distribution of material goods should reflect human differences as measured on all these different scales. Yet, under capitalism, the ability 20 to make money (the green thumb of bourgeois society) enables its possessor to acquire almost every other sort of social good, such as the respect and esteem of others. For the following question, consider each of the choices separately and select all that apply5.The passage provides sufficient information to answer which of the following questions EXCEPT?□A What weight in relation to other qualities should a quality like sensitivity have, according to Walzer, in determining the proper distribution of goods?□B Which quality does Walzer deem too highly valued under liberal capitalism?□C Which are the social goods that are, according to Walzer, outside the reach of the power of money?6. The author implies that Walzers interpretation of the principle of reward according to merit is distinctive for its○A insistence on maximizing everyones rewards○B emphasis on equality○C proven validity○D broad conception of what constitutes merit○E broad conception of what constitutes a reward正确答案:5. A C 6. D.新GRE规律阅读练习题Questions 7 to 9 are based on the following reading passage.National character is not formally considered by social scientists in discussing economic and social development today. They believe that people differ and that these differences should be taken into account 5 somehow, but they have as yet discovered no way to include such variables in their formal models of economic and social development. The difficulty lies in the nature of the data that supposedly define different national characters. Anthropologists and others are on 10 much firmer ground when they attempt to describe the cultural norms for a small homogeneous tribe or village than when they undertake the formidable task of discovering the norms that exist in a complex modern nation-state composed of many disparate groups. The 15 situation is further complicated by the nature of judgments about character; sincesuch judgments are overly dependent on impressions and since, furthermore, impressions are usually stated in qualitative terms, it is impossible to make a reliable comparison between the national characters of two countries.7. The authors main point in the passage is that national character○A is too elusive to merit attention by anthropologists and other social scientists.○B is of greater interest to social scientists today than it has been in the past.○C is still too difficult to describe with the precision required by many social scientists.○D has become increasingly irrelevant because of the complexity of modern lift.○E can be described more accurately by anthropologists than by other social scientists. For the following question, consider each of the choices separately and select all that apply8. It can be inferred from the passage that the social scientists mentioned in the first two sentences would agree with which of the following statements?□A It is extremely difficult to create models that account for both economic and social development□B Models of economic and social development would be improvedby the inclusion of adequate descriptions of national character.□C it is important to supplement formal models of economic and social development with qualitative impressions of national character.9. Which of the following best describes the organization of the passage?○A A problem is presented and reasons for its existence are supplied.○B A controversial view is presented and evidence for its validity is supplied.○C A hypothesis is presented and possible means of verifying it are suggested.○D A recent development is described and then analyzed.○E A dispute is summarized and one side defended.正确答案:7. C 8. B. 9. A新GRE规律阅读练习题Questions 10 and 11 are based on the following reading passage.It is now established that the Milky Way is far more extended and of much greater mass than was hitherto thought. However, all that is visible of the constituents of the Milky Ways corona (outer edge),where much of the galaxys mass must be located, is a tiny fraction of the coronas mass. Thus, most of the Milky Ways outlying matter must be dark. Why? Three facts are salient. First, dwarf galaxies and globular clusters, intowhich most of the stars of the Milky Ways corona are probably bound, consist mainly of old stars. Second, old stars are not highly luminous. Third, no one has detected in the corona the clouds of gaseous matter such as hydrogen and carbon monoxide that are characteristic of the bright parts of a galaxy.10. The passage as a whole is primarily concerned with(A) analyzing a current debate(B) criticizing a well-established theory(C) showing how new facts support a previously dismissed hypothesis(D) stating a conclusion and adducing evidence that may justify it(E) contrasting two types of phenomena and showing how they are related11. Select the sentence that the author implicitly indicates what astronomers believed about the Milky way until fairly recently.正确答案:10. D 11. It is now established...。

GRE阅读真题

GRE阅读真题

GRE阅读真题GRE阅读真题今日我给大家整理了GRE阅读真题之PP2-2,一起来学习吧,下面我就和大家共享,来观赏一下吧。

GRE阅读真题之PP2-2 Passage 1PP2-2Passage 1Objectively, of course, the various ecosystems that sustain life on the planet proceed independently of human agency, just as they operated before the hectic ascendancy of Homo sapiens. But it is also true that it is difficult to think of a single such system that has not, for better of worse, been substantially modified by human culture. Nor is this simply the work of the industrial centuries. It has been happening since the days of ancient Mesopotamia. It is coeval with the origins of writing, and has occurred throughout our social existence. And it is this irreversibly modified world, from the polar caps to the equatorial forests, that is all the nature we have.For the following question, consider each of the choices separately and choose all that apply.1. It can be inferred from the passage that the author would agree with which of the following statements?A. Over tie, the impact of human culture on the natural world has been largely benign.B. It is a mistake to think that the natural world contains many areas of pristine wilderness.C. The only substantial effects that human agency has had on ecosystems have been inadvertent.2. The author mentions “ancient Mesopotamia” primarily in order toA. provide some geographical and historical context for an earlier claim about the ascendancy of Homo sapiensB. support the idea that the impact of human culture on nature was roughly the same in the ancient world as in later timesC. identify a place where the relationship between culture and nature was largely positiveD. emphasize the extent to which the modification of nature by human culture preceded the industrial periodE. make a connection between the origins of writing and other aspects of human cultural developmentGRE阅读真题之PP2-2 Passage 2PP2-2Passage 2As an example of the devastation wrought on music publishers by the photocopier, one executive noted that for a recent choral Festival with 1,200 singers, the festival’s organizing committee purchased only 12 copies of the music published by her company that was performed as part of the festival.1. Which of the following, if true, most seriously weakens the support the example lends to the executive’s contention that music publishers have been devastated by the photocopier?A. Only a third of the 1,200 singers were involved in performing the music published by the executive’s company.B. Half of the singers at the festival had already heard the music they were to perform before they began to practice for the festival.C. Because of shortages in funding, the organizing committee of the choral festival required singers to purchase their own copies of the music performed at the festival.D. Each copy of music that was performed at the festival was shared by two singers.E. As a result of publicity generated by its performance at the festival, the type of music performed at the festival became more widely known.GRE阅读真题之PP2-2 Passage 3PP2-2Passage 3A tall tree can transport a hundred gallons of water a day from its roots deep underground to the treetop. Is this movement propelled by pulling the water from above or pushing it from below? The pull mechanism has long been favored by most scientists. First proposed in the late 1800s, the theory relies on a property of water not commonly associated with fluids: it tensile strength. Instead of making a clean break,water evaporating from treetops tugs on the remaining water molecules, with that tug extending from molecule to molecule all the way down to the roots. The tree itself does not actually push or pull; all the energy for lifting water comes from the sun’s evaporative power.For the following question, consider each of the choices separately and choose all that apply.1. Which of the following statement is supported by the passage?A. The pull theory is not universally accepted by scientists.B. The pull theory depends on one of water’s physical properties.C. The pull theory originated earlier than did the push theory.2. The passage provides information on each of the following EXCEPT:A. when the pull theory originatedB. the amount of water a tall tree can transportC. the significance of water’s tensile strength in the pull theoryD. the role of the sun in the pull theoryE. the mechanism underlying water’s tensile strengthGRE阅读真题之PP2-2 Passage 4PP2-2Passage 4The most plausible justification for higher taxes on automobile fuel is that fuel consumption harms the environment and thus adds to the costs of traffic congestion. But the fact that burning fuel creates these “negative externalities” does not imply that no tax on fuel could ever be too high. Economics is precise about the tax that should, in principle, be levied to deal with negative externalities: the tax on a liter of fuel should be equal to the harm caused by using a liter of fuel. If the tax is more than that, its costs (including the inconvenience to those who would rather have used their cars) will exceed its benefits (including any reduction in congestion and pollution).1. In the context in which it appears, “exceed” most nearly meansA. outstripB. magnifyC. delimitD. offsetE. supplant2. Which of the following best characterizes the function of the indicated portion of the passage?A. It restates a point made earlier in the passage.B. It provides the evidence on which a theory is based.C. It presents a specific application of a general principle.D. It summarizes a justification with which the author disagrees.E. It suggests that the benefits of a particular strategy have been overestimated.GRE阅读真题之PP2答案Passage第一题其次题第三题第四题1BD2C3ABE4ACGRE阅读真题文章到此就结束了,欢迎大家下载使用并丰富,共享给更多有需要的人。

新GRE阅读练习题精选5篇

新GRE阅读练习题精选5篇

新GRE阅读练习题精选5篇为了让同学们更高效的备考GRE阅读,我搜集了新GRE阅读练习题,下面我就和大家共享,来观赏一下吧。

新GRE阅读练习题Factory workers in Beltania are guaranteed lifetime jobs, bonuses paid on thebasis of productivity and corporate profits, and a wage rate that is not attachedto a particular job. Paradoxically, these guarantees do not discourage factoryowners from introducing labor-saving machinery. Such innovations are to theactory owners advantage despite the fact that the owners must protect thewages of their workers.Which of the following, if true, most helps to explain why the introduction of labor-saving machinery is advantageous to factory owners?A Before a Beltanian factory worker is hired, he or she must present a recordof his or her previous productivity.B Labor-saving machinery increases productivity, thus yielding profits thatmore than cover the cost of retraining workers for other jobs.C The purchase and maintenance of new machinery adds significantly to thefinal cost of the goods produced.D Factory workers demand a change of procedure in the routine tasks they perform.E Limited competition exists among Beltanian factories for consumer markets.答案:B新GRE阅读练习题One of the truisms of the advertising industry is that it is rarely necessary to say something of substance in an advertisement in order to boost sales. Instead, one only needs to attract the potential customers attention; memory does the rest, for it is more important for sales that people know of a product than that they know something about it.Which of the following is assumed by the argument?A People can remember a product without having much information about it.B Advertisements, in their own way, function to improve peoples memories.C Attracting a potential customers attention is a simple matter.D The advertising industry knows little of substance about the products it promotes.E Advertisements seldom tell the truth about a product.答案:A新GRE阅读练习题Excluding purchases by businesses, the average amount spent on a factory-new car has risen 30 percent in the last five years. In the average household budget, the proportion spent on car purchases has remained unchanged in that period. Therefore the average household budget must have increased by30 percent over the last five years.Which of the following is an assumption on which the argument relies?(A)The average number of factory-new cars pur-chased per household has remained unchanged over the last five years.(B)The average amount spent per car by businesses buyingfactory-new cars has risen 30 percent in the last five years.(C)The proportion of the average household budget spent on allcar-related expenses has remained unchanged over the last five years.(D)The proportion of the average household budget spent on food and housing has remained unchanged over the last five years.(E)The total amount spent nationwide on factory-new cars has increased by 30 percent over the last five years.答案:A新GRE阅读练习题The bodies of dwarf individuals of mammalian species are generally smaller in relation to those of nondwarf individuals than are the teeth of the dwarf individuals in relation to those of the nondwarf indi-viduals. Fragmentary skeletal remains of an adult dwarf woolly mammoth were recently found. The teeth are three-fourths the size of the teeth of an average adult nondwarf woolly mammoth.The statements above, if true, most strongly support which of the following?(A)The body of the dwarf woolly mammoth was less thanthree-fourths the size of the body of an average adult nondwarf woollymammoth.(B)None of the teeth of the dwarf woolly mammoth that were recently discovered was as large as any of the teeth of nondwarf woolly mammoths that have been discovered.(C)The teeth of most adult dwarf individuals of mammalian species are three- fourths the size of the teeth of the adult nondwarf individuals of the same species.(D)Dwarf woolly mammoths had the same number of teeth as did nondwarf woolly mammoths.(E)Dwarf individuals of most mammalian species are generally no more than three-fourths the size of the adult nondwarf individuals of those species.新GRE阅读练习题Questions 1-4Along a street that is currently without trees, seven trees are to be planted in the pattern1 3 5 72 4 6Where each number designates the position of a tree. No more than two kinds of trees can be planted. If maples are used, no maple can be planted adjacent to or immedi-ately diagonally opposite another maple. Two trees are adjacent to each other if the numbers of their positions differ by two, and immediately diagonally opposite if their numbers differ by one. The following trees, of three kinds, are available for planting:Three red oaksFour maplesFour sycamores1.Which of the following can be the trees planted along the side of the street that has four trees, in order of their positions beginning with position 1?(A)Maple, sycamore, maple, sycamore(B)Maple, sycamore, red oak, maple(C)Red oak, maple, maple, red oak(D)Sycamore, sycamore, maple, maple(E)Sycamore, sycamore, red oak, red oak2.If red oaks are used, then which of the following must be true?(A)The other trees used are all maples.(B)The other trees used are all sycamores.(C)The red oaks are in positions 1, 2, and 3.(D)The red oaks are in positions 3, 4, and 5.(E)The red oaks are in positions 4, 5, and 6.3.Among the trees left over after the planting is done there must be(A)at least one maple(B)at least one red oak(C)at least one sycamore(D)at most one maple(E)at most one red oak4.If maples are planted, the side of the street that has four trees must have(A)red oaks in positions 1 and 7(B)red oaks in positions 3 and 5(C)sycamores in positions 1 and 3(D)sycamores in positions 1 and 7(E)sycamores in positions 3 and 5答案:EB AE。

2024年GRE考试语文真题集锦

2024年GRE考试语文真题集锦

2024年GRE考试语文真题集锦随着时间的推移,考试的形式也在不断变化。

在2024年的GRE考试中,语文作为一项重要的内容,扮演着重要的角色。

本文将为您提供一份2024年GRE考试语文真题集锦,以便您更好地了解考试内容并为备考做好准备。

第一部分:阅读理解阅读理解是GRE考试语文部分的一个重点,考察考生对于文章内容的理解和分析能力。

以下是2024年GRE考试语文真题集锦中的一道阅读理解题目:Passage 1The Industrial Revolution, which began in the 18th century, brought about significant changes in society and the economy. One of the key developments during this period was the invention of the steam engine, which revolutionized transportation and manufacturing. Steam-powered machines allowed for mass production, leading to increased productivity and economic growth.Question:According to the passage, what was one of the key developments during the Industrial Revolution?A) Increased productivityB) Invention of the steam engineC) Economic growthD) Mass production答案:B。

根据文章内容可知,在工业革命期间,蒸汽机的发明是其中一个关键的发展。

GRE阅读36套

GRE阅读36套

GRE阅读36套GRE阅读36套精选1.1许多EB小说WH(艾米丽勃朗特小说《呼啸山庄》)的评论家认为,小说第二部分即使不是逆转了第一部分的内容,也是第一部分的对比物,对第一部分进行了批判评论。

在小说第一部分,浪漫的解读能获得更多的确证。

将两部分视为一个整体受到了小说复杂结构的影响,这种复杂结构体现在小说频繁复杂转换叙述者和时间。

诚然,这些元素出现姑且不必认为是和HJ一样是作者对小说结构的意识,但是他们的出现确实鼓励人试图将小说的不同部分统一起来。

但是,任何试图要将小说所有不同元素统一起来的诠释都注定差强人意。

这倒不是因为这种诠释会必然导致主题刻板僵化(尽管对任何小说的刻板诠释都是危险的),而是因为《呼啸山庄》具有一些难以驾驭的因素,以无可辩驳的力量最终抗拒任何囊括所有的诠释。

在这一点上,《呼啸山庄》和《哈姆雷特》有异曲同工之处。

1.2通常深海中稀少的动物群落都是小虫和甲壳类动物,大型动物更加稀少。

然而,在水热vent,即海中热水从海底处涌出的地方,生活着密度惊人的大型蛤蜊、盲蟹和鱼。

大多数深海动物的食物依赖的特殊物质,最终都来自于浅海落下的来自于光合作用的物质。

然而,要维持这些大的生物群落,食物来源必然要比正常落下的多很多倍。

第一份描述vent动物群落的报告提出了两个可能的食物来源:微生物化学合成,即通过微生物利用化学变化形成的食物,以及水平对流,即从附近区域飘过来的食物。

后来,支持来自本地化学合成的证据越来越多,包括:在这些vent处的水中发现了硫氢化物、许多vent处的微生物具有化学合成能力、以前曾认为是无生物的vent水样中发现了密度很大的微生物。

最后的证据至关重要。

如果这些密度惊人的微生物是vent水流的典型特征,那么vent 处的食物就会使任何来自平流的食物相形见绌。

因此,微生物化学合成物提供了水热vent处的食物链成为被广泛引用的结论,这一结论十分令人振奋,因为地球上还没有其他群落不依赖于光合作用。

gre真题阅读题

gre真题阅读题

gre真题阅读题
1.gre真题总汇
gre阅读题目汇总,gre阅读题目及答案解析汇总请:文章最后一道选项:scannability,一般来说,文章最后两道选项会给出一个选项,但是每道题目都会给出一个选项,那是因为这是在文章的开头部分。

所以,我们要先明白一点,这是一个很好的选项。

我们可以看到,gre的verbal部分有两道题,每一道选项有10多个;每篇10道题有12道题。

每个选项的平均值为一篇文章,所以,在第二道选项时,我们就需要对每个选项的选项有一个大致的了解。

这道题的答案是这样的:这种题一般在文章的开头部分,考生只需要在文章开头或者节选一题之内就可以找到答案。

这道题的第一题就是选择题。

这道题的第一句是选择题的第一句,也就是选择题。

gre真题阅读题 2
gre阅读题目汇总,gre阅读题目及答案解析汇总举个典型的gre阅读题误区。

这个误区误区误区有两面性的,一是阅读不懂二,二是阅读不懂,三是阅读不懂。

gre的阅读题目数量庞大,题目多是单词都认识,不懂,不懂的话题就会出错。

gre 的verbal部分的难点在于单纯的语法,在单词的同时也不需要理解整篇文章。

gre阅读中的词汇题占比不高。

但是如果是考研的同学,考完gre的话,还是需要考一下语法。

gre的verbal部分难度较小。

gre的阅读部分比较难的,一般是词汇量,句子结构,句子结构,长难句的分析。

gre考试的阅读部分比较难,因为词汇量较大,阅读速度跟上文章节节奏都不一样。

所以考生要先攻克gre长难句。

GRE之OG2阅读真题精选

GRE之OG2阅读真题精选

GRE之OG2阅读真题精选GRE阅读真题之OG2 Passage 31OG-2Passage 31Recently an unusually high number of dolphins have been found dead of infectious diseases, and most of these had abnormally high tissue concentrations of certain compounds that, even in low concentrations, reduce dolphins’ resistance to infection. The only source of these compounds in the dolphins’ environment is boat paint. Therefore, since dolphins rid their bodies of the compounds rapidly once exposure ceases, their mortality rate should decline rapidly if such boat paints are banned.1. Which of the following, if true, most strengthens the argument?A. The levels of the compounds typically used in boat paints today are lower than they were in boat paints manufactured a decade ago.B. In high concentrations, the compounds are toxic to many types of marine animals.C. The compounds break down into harmless substances after a few months of exposure to water or air.D. High tissue levels of the compounds have recently been found in some marine animals, but there is no record of any of those animals dying in unusually large numbers recently.E. The compounds do not leach out of the boat paint if the paint is applied exactly in accordance with the manufacturer’s directions.GRE阅读真题之OG2 Passage 32OG-2Passage 32The work of English writer Aphra Behn (1640–1689) changed markedly during the 1680s, as she turned from writing plays to writing prose narratives. According to literary critic Rachel Carnell, most scholars view this change as primarily motivated by financial considerations: earning a living by writing for the theatre became more difficult in the 1680s, so Behn tried various other types of prose genres in the hope of finding another lucrative medium. In fact, a long epistolary scandal novel that she wrote in the mid-1680s sold quite well. Yet, as Carnell notes, Behn did not repeat this approach in her other prose works; instead, she turned towriting shorter, more serious novels, even though only about half of these were published during her lifetime. Carnell argues that Behn, whose stage productions are primarily comedies, may have turned to an emerging literary form, the novel, in a conscious attempt to criticize, and subvert for her own ends, the conventions and ideology of a well-established form of her day, the dramatic tragedy.Carnell acknowledges that Behn admired the skill of such contemporary writers of dramatic tragedy as John Dryden, and that Behn’s own comic stage productions displayed the same partisanship for the reigning Stuart monarchy that characterized most of the politically oriented dramatic tragedies of her day. However, Carnell argues that Behn took issue with the way in which these writers and plays defined the nature of tragedy. As prescribed by Dryden, tragedy was supposed to concern a heroic man who is a public figure and who undergoes a fall that evokes pity from the audience. Carnell points out that Behn’s tragic novels focus instead on the plight of little-known women and the private world of the household; even in her few novels featuring male protagonists, Behn insists on the importance of the crimes these otherwise heroic figures commit in the domestic sphere.Moreover, according to Carnell, Behn questioned the view promulgated by monarchist dramatic tragedies such as Dryden’s: that the envisioned “public” political ideal—passive obedience to the nation’s king—ought to be mirrored in the private sphere, with family members wholly obedient to a male head of household. Carnell sees Behn’s novels not only as rejecting the model of patriarchal and hierarchical family order, but also as warning that insisting on such a parallel can result in real tragedy befalling the members of the domestic sphere. According to Carnell, Behn’s choice of literary form underscores the differences between her own approach to crafting a tragic story and that taken in the dramatic tragedies, with their artificial distinction between the public and private spheres. Behn’s novels engage in the political dialogue of her era by demonstrating that the good of the nation ultimately encompasses more than the good of the public figures who rule it.1. The passage is primarily concerned withA. tracing how Behn’s view of the nature of tragedy changed over timeB. explaining one author’s view of Behn’s contribu tion to the development of an emerging literary formC. differentiating between the early and the late literary works of BehnD. contrasting the approaches to tragedy taken by Behn and by DrydenE. presenting one scholar’s explanation for a maj or development in Behn’s literary career2. The passage suggests that Carnell sees Behn’s novels featuring male protagonists as differing from dramatic tragedies such as Dryden’s featuring male protagonists in that the formerA. depict these characters as less than heroic in their public actionsB. emphasize the consequences of these characters’ actions in the private sphereC. insist on a parallel between the public and the private spheresD. are aimed at a predominantly female audienceE. depict family members who disobey these protagonists3. The passage suggests that Carnell believes Behn held which of the following attitudes about the relationship between the private and public spheres?A. The private sphere is more appropriate than is the publicsphere as the setting for plays about political events.B. The structure of the private sphere should not replicate the hierarchical order of the public sphere.C. Actions in the private sphere are more fundamental to ensuring the good of the nation than are actions in the public sphere.D. Crimes committed in the private sphere are likely to cause tragedy in the public sphere rather than vice versa.E. The private sphere is the mirror in which issues affecting the public sphere can most clearly be seen.4. It can be inferred from the passage that the “artificial distinction” (line 53-54) refers to theA. practice utilized in dramatic tragedies of providing different structural models for the public and the private spheresB. ideology of many dramatic tragedies that advocate passive obedience only in the private sphere and not in the public sphereC. convention that drama ought to concern events in the public sphere and that novels ought to concern events in the private sphereD. assumption made by the authors of conventionaldramatic tragedies that legitimate tragic action occurs only in the public sphereE. approach taken by the dramatic tragedies in depicting male and female characters differently, depending on whether their roles were public or privateGRE阅读真题之OG2 Passage 33OG-2Passage 33Computers cannot accurately predict climate change unless the mathematical equations fed into them adequately capture the natural meteorological processes they are intended to simulate. Moreover, there are processes that influence climate, such as modifications in land use, that scientists do not know how to simulate. The failure to incorporate such a process into a computer climate model can lead the model astray because a small initial effect can initiate a feedback cycle: a perturbation in one variable modifies a second variable, which in turn amplifies the original disturbance. An increase in temperature, for example, can boost the moisture content of the atmosphere, which then causes further warming because water vapor is a greenhouse gas.For the following question, consider each of the choicesseparately and choose all that apply.1. The passage mentions which of the following as adversely affecting the accuracy of computer predictions of climate change?A. Failure to allow for some of the processes that influence climateB. Mathematical equations that do not accurately reflect natural phenomenaC. An overestimate of the role of feedback cycles2. In the context in which it appears, “amplifies” (line 11) most nearly meansA. exacerbatesB. explicatesC. expatiatesD. adds detail toE. makes louderGRE阅读真题之OG2 Passage 34OG-2Passage 34Extensive housing construction is underway in Pataska Forest, the habitat of a large population of deer. Because deer feed at the edges of forests, these deer will be attracted to the spacesalongside the new roads being cut through Pataska Forest to serve the new residential areas. Consequently, once the housing is occupied, the annual number of the forest’s deer hit by cars will be much higher than before construction started.1. Which of the following is an assumption on which the argument depends?A. The number of deer hit by commercial vehicles will not increase significantly when the housing is occupied.B. Deer will be as attracted to the forest edge around new houses as to the forest edge alongside roads.C. In years past, the annual number of deer that have been hit by cars on existing roads through Pataska Forest has been very low.D. The development will leave sufficient forest to sustain a significant population of deer.E. No deer hunting will be allowed in Pataska Forest when the housing is occupied.GRE阅读真题之OG2 Passage 35OG-2Passage 35While chocolate was highly esteemed in Mesoamerica,where it originated, its adoption in Europe was initially slow. There is a common belief that Europeans needed t o “transform” chocolate to make it appetizing. However, while Spaniards did put sugar, which was unknown to indigenous Americans, into chocolate beverages, this additive was not completely innovative. Mesoamericans were already sweetening chocolate with honey, and the step from honey to sugar—increasingly more available than honey because of expanding sugar plantations in the Americas—is a small one. Likewise, although Spaniards adjusted Mesoamerican recipes by using European spices, the spices chosen suggest an attempt to replicate harder-to-find native flowers. There is no indication the Spaniards deliberately tried to change the original flavor of chocolate.1. The author of the passage refers to the use of honey primarily toA. identify the origins of an additive previously untried by EuropeansB. present an example of a product that was unknown to EuropeansC. correct the misapprehension that Mesoamericans used a sweetener that was not available in EuropeD. provide an example of an ingredient that was in the process of being displaced by a substituteE. explain why the Spanish use of sugar in chocolate was nota sign of a need to transform chocolate2. Which sentence presents a misconception that the passage challenges?A. The second (“There is 。

GRE之OG2阅读真题汇总

GRE之OG2阅读真题汇总

GRE之OG2阅读真题汇总GRE阅读真题之OG2OG-2Passage 6The more definitions a given noun has, the more valuable is each one. Multiple definitions, each subtly different from all the others, convey multiple shades of meaning. They expand the uses of the word; language is enriched, thought is widened, and interpretations increase or dilate to fill the potentialities of association. The very impossibility of absoluteness in the definition of certain nouns adds to the levels of connotation they may reach. The inner life of a writer often says more than most readers can know; the mind of a reader can discover truths that go beyond the intent or perhaps even the comprehension of the writer. And all of it finds expression because a word can mean many things.1. In the context in which it appears, “shades” (line 2) most nearly meansA. remindersB. nuancesC. obscuritiesD. coveringsE. degrees2. The passage suggests that a writer’s use of nouns that have multiple definitions can have which of the following effects on the relationship between writer and reader?A. It can encourage the reader to consider how the writer’s life might have influenced the work.B. It can cause the reader to become frustrated with the writer’s failure to distinguish between subtle shades of meaning.C. It can allow the reader to discern in a work certain meanings that the writer did not foresee.D. It allows the writer to provide the reader with clues beyond the word itself in order to avoid ambiguity.E. It allows the writer to present unfamiliar ideas to the reader more efficiently.GRE阅读真题之OG2OG-2Passage 7Until recently, many anthropologists assumed that the environment of what is now the southwestern United States shaped the social history and culture of the region’s indi genous peoples. Building on this assumption, archaeologists asserted that adverse environmental conditions and droughts were responsible for the disappearances and migrations ofsouthwestern populations from many sites they once inhabited.However, such deterministic arguments fail to acknowledge that local environmental variability in the Southwest makes generalizing about that environment difficult. To examine the relationship between environmental variation and sociocultural change in the Western Pueblo region of central Arizona, which indigenous tribes have occupied continuously for at least 800 years, a research team recently reconstructed the climatic, vegetational, and erosional cycles of past centuries. The researchers found it impossible to provide a single, generally applicable characterization of environmental conditions for the region. Rather, they found that local areas experienced different patterns of rainfall, wind, and erosion, and that such conditions had prevailed in the Southwest for the last 1,400 years. Rainfall, for example, varied within and between local valley systems, so that even adjacent agricultural fields can produce significantly different yields.The researchers characterized episodes of variation in southwestern environments by frequency: low-frequency environmental processes occur in cycles longer than one human generation, which generally is considered to last about 25 years, and high-frequency processes have shorter cycles. The researchers pointed out that low-frequency processes, such as fluctuations in stream flow and groundwater levels, would not usually be apparent to human populations. In contrast, high-frequency fluctuationssuch as seasonal temperature variations are observable and somewhat predictable, so that groups could have adapted their behaviors accordingly. When the researchers compared sequences of sociocultural change in the Western Pueblo region with episodes of low- and high-frequency environmental variation, however, they found no simple correlation between environmental process and sociocultural change or persistence.Although early Pueblo peoples did protect themselves against environmental risk and uncertainty, they responded variously on different occasions to similar patterns of high-frequency climatic and environmental change. The researchers identified seven major adaptive responses, including increased mobility, relocation of permanent settlements, changes in subsistence foods, and reliance on trade with other groups. These findings sugges t that groups’ adaptive choices depended on cultural and social as well as environmental factors and were flexible strategies rather than uncomplicated reactions to environmental change. Environmental conditions mattered, but they were rarely, if ever, sufficient to account for sociocultural persistence and change. Group size and composition, culture, contact with other groups, and individual choices and actions were —barring catastrophes such as floods or earthquakes — more significant fora population’s survival than were climate and environment.1. The passage is primarily concerned withA. explaining why certain research findings have created controversyB. pointing out the flaws in a research methodology and suggesting a different approachC. presenting evidence to challenge an explanation and offering an alternative explanationD. elucidating the means by which certain groups have adapted to their environmentE. defending a long-held interpretation by presenting new research findings2. Which of the following findings would most strongly support the assertion made by the archaeologists mentioned in line 3?A. A population remained in a certain region at least a century after erosion wore away much of the topsoil that sustained grass for their grazing animals.B. The range of a certain group’s agricultural activity increased over a century of gradual decrease in annual rainfall.C. As winters grew increasingly mild in a certain region, the nomadic residents of the region continued to move between their summer and winter encampments.D. An agricultural population began to trade for supplies of a grain instead of producing the grain in its own fields as it had in the past.E. A half century of drought and falling groundwater levels caused acertain population to abandon their settlements along a riverbank.3. The fact that “adjacent agricultural fields can produce significantly different yields” (lines 16–17) is offered as evidence of theA. unpredictability of the climate and environment of the southwestern United StatesB. difficulty of producing a consistent food supply for a large population in the Western Pueblo regionC. lack of water and land suitable for cultivation in central ArizonaD. local climatic variation in the environment of the southwestern United StatesE. high-frequency environmental processes at work in the southwestern United States4. It can be inferred from the passage that which of the following activities is NOT an example of a population responding to high-frequency environmental processes?A. Developing watertight jars in which to collect and store water during the rainy seasonB. Building multistory dwellings in low-lying areas to avoid the flash flooding that occurs each summerC. Moving a village because groundwater levels have changed over the last generationD. Trading with other groups for furs from which to make winterclothesE. Moving one’s herds of grazing animals each year between summer and winter pasturesGRE阅读真题之OG2OG-2Passage 8Arctic sea ice comes in two varieties. Seasonal ice forms in winter and then melts in summer, while perennial ice persists year-round. To the untrained eye, all sea ice looks similar, but by licking it, one can estimate how long a particular piece has been floating round. When ice begins to form in seawater, it forces out salt, which has no place in the crystal structure. As the ice gets thicker, the rejected salt collects in tiny pockets of brine too highly concentrated to freeze. A piece of first-year ice will taste salty. Eventually, if the ice survives, these pockets of brine drain out through fine, veinlike channels, and the ice becomes fresher; multiyear ice can even be melted and drunk.For the following question, consider each of the choices separately and select all that apply.1. The passage mentions which of the following as being a characteristic of seasonal ice?A. It is similar in appearance to perennial ice.B. It is typically filled with fine, veinlike channels.C. It tastes saltier than perennial ice.2. In the context in which it appears, “fine” (line 7) most nearly meansA. acceptableB. elegantC. preciseD. pureE. smallGRE阅读真题之OG2OG-2Passage 9Historians credit repeated locust invasions in the nineteenth century with reshaping United States agriculture west of the Mississippi River. Admonished by government entomologists, farmers began to diversify. Wheat had come to nearly monopolize the region, but it was particularly vulnerable to the locusts. In 1873, just before the locusts’ most withering offensive, nearly two-thirds of Minnesota farmland was producing wheat; by the invasions’ last year, that fraction h ad dropped to less than one-sixth. Farmers learned that peas and beans were far less vulnerable to the insects, and corn was a more robust grain than wheat. In addition to planting alternative crops, many farmers turned to dairy and beef production. Although pastures were often damaged by the locusts, these lands were almost always left in better shape than the crops were.For the following question, consider each of the choices separately and select all that apply.1. According to the passage, before the recommendations by the government entomologists, which of the following was true about farming west of the Mississippi River?A. Farmers focused primarily on growing wheat.B. Peas and beans had not yet been planted in the region.C. A relatively small portion of farmland was devoted to crops other than wheat.2. In the context in which it appears, “robust” (line 8) most nearly meansA. crudeB. demandingC. productiveD. vigorousE. richGRE阅读真题之OG2OG-2Passage 10In 1998 the United States Department of Transportation received nearly 10,000 consumer complaints about airlines; in 1999 it received over 20,000. Moreover, the number of complaints per 100,000 passengers alsomore than doubled. In both years the vast majority of complaints concerned flight delays, cancellations, mishandled baggage, and customer service. Clearly, therefore, despite the United States airline industry’s serious efforts to improve performance in these areas, passenger dissatisfaction with airline service increased significantly in 1999.1. Which of the following, if true, most seriously weakens the argument?A. Although the percentage of flights that arrived on time dropped slightly overall, from 77 percent in 1998 to 76 percent in 1999, some United States airlines’ 1999 on-time rate was actually better than their 1998 on-time rate.B. The number of passengers flying on United States airlines was significantly higher in 1999 than in 1998.C. Fewer bags per 1,000 passengers flying on United States airlines were lost or delayed in 1999 than in 1998.D. The appearance in 1999 of many new Internet sites that relay complaints directly to the Department of Transportation has made filing a complaint about airlines much easier for consumers than ever before.E. Although the number of consumer complaints increased for every major United States airline in 1999, for some airlines the extent of the increase was substantial, whereas for others it was extremely small.。

GRE考试《阅读理解》练习题及答案(1)

GRE考试《阅读理解》练习题及答案(1)

GRE考试《阅读理解》练习题及答案(1)GRE阅读题目解析:治疗胃酸的药Columnist: Until very recently, Presorbin and Veltrex, two medications used to block excess stomach acid, were both available only with a prescription wri tten by a doctor. In an advertisement for Presorbin, its makers argue that Pres orbin is superior on the grounds that doctors have written 200 million prescrip tions for Presorbin, as compared to 100 million for Veltrex. It can be argued t hat the number of prescriptions written is never a worthwhile criterion for com paring the merits of medicines, but that the advertisement’s argument is absur d is quite adequately revealed by observing that Presorbin was available as a p rescription medicine years before Veltrex was.1. In the columnist’s argument, the two highlighted portions play which of the following roles?A. The first is a claim that the columnist’s argument seeks to clarify; th e second states a conclusion drawn about one possible interpretation of that cl aim.B. The first identifies the conclusion of an argument that the columnist’s argument is directed against; the second states the main conclusion of the col umnist’s argument.C. The first states the main conclusion of the columnist’s argument; the s econd states a conclusion that the columnist draws in defending that conclusion against an objection.D. The first identifies an assumption made in an argument that the columnis t's argument is directed against; the second states the main conclusion of the columnist’s argument.E. The first is a claim that has been offered as evidence to support a posi tion that the columnist opposes; the second states the main conclusion of the c olumnist’s argument.1Columnist: Until very recently, Presorbin and Veltrex, two medications used to block excess stomach acid, were both available only with a prescription wri tten by a doctor.专栏作者:直到最近,P 和 V,两种治疗胃酸过多的药,都是处方药。

  1. 1、下载文档前请自行甄别文档内容的完整性,平台不提供额外的编辑、内容补充、找答案等附加服务。
  2. 2、"仅部分预览"的文档,不可在线预览部分如存在完整性等问题,可反馈申请退款(可完整预览的文档不适用该条件!)。
  3. 3、如文档侵犯您的权益,请联系客服反馈,我们会尽快为您处理(人工客服工作时间:9:00-18:30)。

使用说明 (1)GRE阅读(No. 2—No. 9) (2)No. 2-1 (2)SECTION A (2)SECTION B (5)No. 2-2 (9)SECTION A (9)SECTION B (12)No. 2-3 (16)SECTION A (16)SECTION B (20)No. 3-1 (23)SECTION A (23)SECTION B (27)No. 3-2 (30)SECTION A (30)SECTION B (34)No. 3-3 (38)SECTION A (38)SECTION B (41)No. 4-1 (45)SECTION A (45)SECTION B (49)No. 4-2 (52)SECTION A (53)SECTION B (56)No. 4-3 (60)SECTION A (60)SECTION B (64)No. 5-1 (68)SECTION A (68)SECTION B (71)No. 5-2 (75)SECTION A (75)SECTION B (78)No. 5-3 (82)SECTION A (82)SECTION B (86)No. 6-1 (89)SECTION B (93)No. 6-2 (97)SECTION A (97)SECTION B (101)No. 6-3 (105)SECTION A (105)SECTION B (108)No. 7-1 (112)SECTION A (112)SECTION B (116)No. 7-2 (120)SECTION A (120)SECTION B (123)No. 7-3 (127)SECTION A (127)SECTION B (131)No. 8-1 (135)SECTION A (135)SECTION B (139)No. 8-2 (143)SECTION A (143)SECTION B (147)No. 8-3 (150)SECTION A (150)SECTION B (154)No. 9-1 (157)SECTION A (157)SECTION B (161)No. 9-2 (165)SECTION A (165)SECTION B (169)No. 9-3 (172)SECTION A (172)SECTION B (176)No. 9-4 (180)SECTION A (180)SECTION B (184)No. 9-5 (187)SECTION A (187)No. 9-6 (195)SECTION A (195)SECTION B (199)GRE国内题全部阅读 (202)1990年04月 (202)SECTION A (202)SECTION B (206)1990年10月 (210)SECTION A (210)SECTION B (214)1991年02月 (218)SECTION A (218)SECTION B (222)1991年04月 (226)SECTION A (226)SECTION B (230)1991年10月 (234)SECTION A (234)SECTION B (237)1992年02月 (241)SECTION A (241)SECTION B (245)1992年04月 (249)SECTION A (249)SECTION B (253)1992年10月 (256)SECTION A (256)SECTION B (260)1993年02月 (264)SECTION A (264)SECTION B (267)1993年04月 (271)SECTION A (271)SECTION B (274)SECTION C (278)1993年10月 (282)SECTION A (282)SECTION B (286)1994年02月 (289)SECTION B (293)1994年04月 (296)SECTION A (296)SECTION B (300)1994年10月 (304)SECTION A (304)SECTION B (308)1995年04月 (312)SECTION A (312)SECTION B (317)1995年10月 (321)SECTION A (321)SECTION B (325)1996年04月北美 (328)SECTION A (328)SECTION B (332)1996年04月 (336)SECTION A (336)SECTION B (340)1996年10月 (344)SECTION A (344)SECTION B (348)1997年04月 (351)SECTION A (351)SECTION B (355)1997年11月 (359)SECTION A (359)SECTION B (363)1998年04月 (366)SECTION A (366)SECTION B (370)1998年11月 (374)SECTION A (374)SECTION B (378)1999年04月 (382)SECTION A (382)SECTION B (386)使用说明1、各题答案均隐藏在(D)选项后(放在这里主要是避免下一题答案也被显示从而影响做题),另外文件最后有全部答案(感谢PUMPKIN的辛勤劳动!),显示的方法是鼠标左键单击“常用”工具栏“显示/隐藏编辑标记”按钮(只有WORD才有此按钮,写字板没有)。

再次单击又被隐藏。

见下图:2、做题时请切换至“普通”视图,因为显示答案时页面变化小;在“页面”视图下,可以欣赏到以当今世界一流技术精心打造的一流风姿!3、本文件包括所有的GMAT阅读、GRE全部No题和99年之前全部国内题的阅读以及(28+1)套LSAT阅读,共计2772题,是无比宝贵的学习材料(也是馈赠亲友的最珍贵礼物,并值得永久收藏)。

它不仅仅能够提高阅读能力,也是扩大词汇、研究语法的宝贵材料。

GRE的难度突出表现在语言文字的艰深和意思的晦涩,而LSAT题对逻辑思维的要求很高,GMA T文字上的难度不如GRE,逻辑上次于LSAT(个人观点)。

4、文档中绿色单词或词组的后面有释义,看注释请按上述第1点操作。

少数难题加注了我个人的看法(一律被隐藏),未必正确,仅供参考。

5、一部分文档题目中的行号添加了超链接,按住Ctrl的同时用鼠标左键单击有超链接标志的行号即可到达相应内容。

但因为我时间紧,没有做完(对于没有超链接的题目,做题时可以用“查找”命令搜索。

无论如何,比纸面的用起来肯定方便。

在电脑上做题吧,如果你的WORD用的比较熟练,再在你的电脑上装几本电子词典,你会感到效率倍增,妙趣无穷!);另外,由于WORD软件本身的缺陷,格式上可能未必有“GMAT&LSAT CR”与“GMA T SC 885”那么完美统一。

这些都有待于将来修正与完善。

GRE阅读(No. 2—No. 9)No. 2-1SECTION AExtended debate concerning the exact point of origin of individual folktales told byAfro-American slaves has unfortunately taken precedence over analysis of the tales’ meaning and function. Cultural continuities with Africa were not dependent on importation and perpetuation of specific folktales in their pristine form. It is in the place that tales occupied in the lives of the slaves and in the meaning slaves derived from them that the clearest resemblances to African tradition can be found. Afro-American slaves did not borrow tales indiscriminately from the Whites among whom they lived. Black people were most influenced by those Euro-American tales whose functional meaning and aesthetic appeal had the greatest similarity to the tales with deep roots in their ancestral homeland. Regardless of where slave tales came from, the essential point is that, with respect to language, delivery, details of characterization, and plot, slaves quickly made them their own.17. The author claims that most studies of folktales told by Afro-American slaves areinadequate because the studies(A) fail to recognize any possible Euro-American influence on the folktales(B) do not pay enough attention to the features of a folktale that best reveal anAfrican influence(C) overestimate the number of folktales brought from Africa by the slaves(D) do not consider the fact that a folktale can be changed as it is retold manytimes(E) oversimplify the diverse and complex traditions of the slaves ancestralhomeland18. The author’s main purpose is to(A) create a new field of study(B) discredit an existing field of study(C) change the focus of a field of study(D) transplant scholarly techniques from one field of study to another(E) restrict the scope of a burgeoning new field of study19. The passage suggests that the author would regard which of the following areasof inquiry as most likely to reveal the slaves’ cultural continuities with Africa?(A) The means by which Blacks disseminated their folktales innineteenth-century America(B) Specific regional differences in the styles of delivery used by the slaves intelling folktales(C) The functional meaning of Black folktales in the lives of White childrenraised by slave(D) The specific way the slaves used folktales to impart moral teaching to theirchildren(E) The complexities of plot that appear most frequently in the slaves’ tales20. Which of the following techniques is used by the author in developing theargument in the passage?(A) Giving a cliché a new meaning(B) Pointedly refusing to define key terms(C) Alternately presenting generalities and concrete details(D) Concluding the passage with a restatement of the first point made in thepassage(E) Juxtaposing statements of what is not the case and statements of what is thecaseT he energy contained in rock within the earth’s crust represents a nearly unlimited energy source, but until recently commercial retrieval has been limited to underground hot water and/or steam recovery systems. These systems have been developed in areas of recent volcanic activity, where high rates of heat flow cause visible eruption of water in the form of geysers and hot springs. In other areas, however, hot rock also exists near the surface but there is insufficient water present to produce eruptive phenomena. Thus a potential hot dry rock (HDR) reservoir exists whenever the amount of spontaneously produced geothermal fluid has been judged inadequate for existing commercial systems.As a result of recent energy crisis, new concepts for creating HDR recovery systems—which involve drilling holes and connecting them to artificial reservoirs placed deep within the crust—are being developed. In all attempts to retrieve energy f rom HDR’s, artificial stimulation will be required to create either sufficient permeability or bounded flow paths to facilitate the removal of heat by circulation of a fluid over the surface of the rock.The HDR resource base is generally defined to included crustal rock that is hotter than 150℃, is at depths less than ten kilometers, and can be drilled with presently available equipment. Although wells deeper than ten kilometers are technically feasible, prevailing economic factors will obviously determine the commercial feasibility of wells at such depths. Rock temperatures as low as 100℃may be useful for space heating; however, for producing electricity, temperatures greater than 200℃are desirable.The geothermal gradient, which specifically determines the depth of drilling required to reach a desired temperature, is a major factor in the recoverability of geothermal resources. Temperature gradient maps generated from oil and gas well temperature-depth records kept by the American Association of Petroleum Geologists suggest that tappable high-temperature gradients are distributed all across the United States. (There are many areas, however, for which no temperature gradient records exist.)Indications are that the HDR resource base is very large. If an average geothermal temperature gradient of 22℃per kilometer of depth is used, a staggering 13,000,000quadrillion B.T.U.’s of total energy are calculated to be contained in crustal rock to aten-kilometer depth in the United States. If we conservatively estimate that only about 0.2 percent is recoverable, we find a total of all the coal remaining in the United States. The remaining problem is to balance the economics of deeper, hotter, more costly wells and shallower, cooler, less expensive wells against the value of the final product, electricity and/or heat.21. The primary purpose of the passage is to(A) alert readers to the existence of HDR’s as an available energy source(B) document the challengers that have been surmounted in the effort to recoverenergy from HDR’s(C) warn the users of coal and oil that HDR’s are not an economically feasiblealternative(D) encourage the use of new techniques for the recovery of energy fromunderground hot water and steam(E) urge consumers to demand quicker development of HDR resources for theproduction of energy22. The passage would be most likely to appear in a(A) petrological research report focused on the history of temperature-depthrecords in the United States(B) congressional report urging the conservation of oil and natural gas reserves inthe United States(C) technical journal article concerned with the recoverability of newly identifiedenergy sources(D) consumer report describing the extent and accessibility of remaining coalresources(E) pamphlet designed to introduce homeowners to the advantages of HDRspace-heating systems23. According the passage, an average geothermal gradient of 22℃per kilometer ofdepth can be used to(A) balance the economics of HDR energy retrieval against that of undergroundhot water or steam recovery systems(B) determine the amount of energy that will used for space heating in the UnitedStates(C) provide comparisons between hot water and HDR energy sources in UnitedStates(D) revise the estimates on the extent of remaining coal resources in the UnitedStates(E) estimate the total HDR resource base in the United States24. It can be inferred from the passage that the availability of temperature-depthrecords for any specific area in the United States depends primarily on the(A) possibility that HDR’s may be found in that area(B) existence of previous attempts to obtain oil or gas in that area(C) history of successful hot water or steam recovery efforts in that area(D) failure of inhabitants to conserve oil gas reserves in that area(E) use of coal as a substitute for oil or gas in that area25. According to the passage, in all HDR recovery systems fluid will be necessary inorder to allow(A) sufficient permeability(B) artificial stimulation(C) drilling of holes(D) construction of reservoirs(E) transfer of heat26. According to the passage, if the average geothermal gradient in an area is 22℃per kilometer of depth, which of the following can be reliably predicted?I. The temperature at the base of a 10-kilometer well will be sufficient for theproduction of electricity.II. Drilling of wells deeper than 10 kilometers will be economically feasible.III. Insufficient water is present to produce eruptive phenomena.(A) I only(B) II only(C) I and II only(D) II and III only(E) I, II, and III27. Which of the following would be the most appropriate title for the passage?(A) Energy from Water Sources: The Feasibility of Commercial Systems(B) Geothermal Energy Retrieval: V olcanic Activity and Hot Dry Rocks(C) Energy Underground: Geothermal Sources Give Way to Fossil Fuels(D) Tappable Energy for America’s Future: Hot Dry Rocks(E) High Geothermal Gradients in the United States: Myth or Reality?SECTION BFour legal approaches may be followed in attempting to channel technological development in socially useful direction: specific directives, market incentive modifications, criminal prohibitions, and changes in decision-making structures. Specific directives involve the government’s identifying one or more factors controlling research, development, or implementation of a given technology. Directives affecting such factors may vary from administrative regulation of private activity to government ownership of a technologicaloperation. Market incentive modifications are deliberate alterations of the market within which private decisions regarding the development and implementation of technology are made. Such modifications may consist of imposing taxes to cover the costs to society of a given technology, granting subsidies to pay for social benefits of a technology, creating the right to sue to prevent certain technological development, or easing procedural rules to enable the recovery of damages to compensate for harm caused by destructive technological activity. Criminal prohibitions may modify technological activity in areas impinging on fundamental social values, or they may modify human behavior likely to result from technological applications—for example, the deactivation of automotive pollution control devices in order to improve vehicle performance. Alteration of decision-making structures includes all possible modifications in the authority, constitution, or responsibility of private and public entities deciding questions of technological development and implementation. Such alterations include the addition of public-interest members to corporate boards, the imposition by statute of duties on governmental decision-makers, and the extension of warranties in response to consumer action.Effective use of these methods to control technology depends on whether or not the goal of regulation is the optimal allocation of resources. When the object is optimal resource allocation, that combination of legal methods should be used that most nearly yields the allocation that would exist if there were no external costs resulting from allocating resources through market activity. There are external costs when the price set by buyers and sellers of goods fails to include some costs, to anyone, that result from the production and use of the goods. Such costs are internalized when buyers pay them.Air pollution from motor vehicles imposes external costs on all those exposed to it, in the form of soiling, materials damage, and disease: these externalities result from failure to place a price on air, thus making it a free good, common to all. Such externalities lead to nonoptimal resource allocation, because the private net product and the social net product of market activity are not often identical. If all externalities were internalized, transactions would occur until bargaining could no longer improve the situation, thus giving an optimal allocation of resources at a given time.17. The passage is primarily concerned with describing(A) objectives and legal method for directing technological development(B) technical approaches to the problem of controlling market activity(C) economic procedures for facilitating transactions between buyers and sellers(D) reasons for slowing the technological development in light ofenvironmentalist objections(E) technological innovations making it possible to achieve optimum allocation ofresources18. The author cites air pollution from motor vehicles in lines 54-56 in order to(A) revise cost estimates calculated by including the costs of resources(B) evaluate legal methods used to prevent technological developments(C) give examples of costs not included in buyer-seller bargains(D) refute hypotheses not made on the basis of monetary exchange values(E) commend technological research undertaken for the common welfare19. According to the passage, transactions between private buyers and sellers haveeffects on society that generally(A) are harmful when all factors are considered(B) give rise to ever-increasing resource costs(C) reflect an optimal allocation of natural resources(D) encompass more than the effects on the buyers and sellers alone(E) are guided by legal controls on the development of technology20. It can be inferred from the passage that the author does NOT favor which of thefollowing?(A) Protecting the environment for future use(B) Changing the balance of power between opposing interests in business(C) Intervening in the activity of the free market(D) Making prices reflect costs to everyone in society(E) Causing technological development to cease21. A gasoline-conservation tax on the purchase of large automobiles, with theproceeds of the tax rebated to purchasers of small automobiles, is an example of(A) a specific directive(B) a market incentive modification(C) an optimal resource allocation(D) an alteration of a decision-making structure(E) an external cost22. If there were no external costs, as they are described in the passage, which of thefollowing would be true?(A) All technology-control methods would be effective.(B) Some resource allocations would be illegal.(C) Prices would include all costs to members of society.(D) Some decision-making structures would be altered.(E) The availability of common goods would increase.23. The author assumes that, in determining what would be an optimal allocation ofresources, it would be possible to(A) assign monetary value to all damage resulting from the use of technology(B) combine legal methods to yield the theoretical optimum(C) convince buyers to bear the burden of damage from technologicaldevelopments(D) predict the costs of new technological developments(E) derive an equation making costs depend on prices24. On the basis of the passage, it can be inferred that the author would agree withwhich of the following statements concerning technological development?(A) The government should own technological operations.(B) The effect of technological development cannot be controlled.(C) Some technological developments are beneficial.(D) The current state of technological development results in a good allocation ofresources.(E) Applications of technological developments are criminally destructive.The whole biosphere, like the individual organisms that live inside it, exists in a chemically dynamic state. In this homeostatic system, a great number of organic compounds are synthesized, transformed, and decomposed continuously; together, these processes constitute the major parts of the carbon cycle. For the smooth operation of this cycle, degradation is just as important as synthesis: the green plants produce great quantities of polymers, such as cellulose, and innumerable other compounds like alkaloids, terpenes, and flavonoids, that green plants cannot use as sources of energy during respiration. The release of the carbon in these compounds for recycling depends almost entirely on the action of both aerobic and anaerobic bacteria and certain types of fungi. Some bacteria and fungi possess the unique and extremely important biochemical asset of being able to catalyze the oxidation of numerous inert products, thereby initiating reaction sequences that produce carbon dioxide and so return much carbon to a form that actively enters into life cycles once again.25. The passage contains information that would answer which of the followingquestions about the carbon cycle?I. What are some of the compounds that are broken down in the carbon cycle?II. Why are some compounds that are involved in the carbon cycle less reactive than others?III. What role do bacteria and fungi play in the carbon cycle?(A) I only(B) II only(C) III only(D) I and II only(E) I and III only26. The author implies that which of the following is the primary reason thatdegradation is as important as synthesis to the smooth operation of the carboncycle?(A) Most of the polymers and organic compounds found in the plant kingdom arechemically unstable.(B) The synthesis of some organic material deprives life processes of an energysource.(C) Decomposition permits the recycling of carbon that would otherwise be fixedin certain substances.(D) Many organisms cannot use plants as a source of food, but can feed onbacteria and fungi.(E) Bacteria and fungi could not survive if some carbon compounds were notdegraded.27. The author’s contention about the importance of bacteria and fungi in theproduction of energy for life processes would be most clearly strengthened ifwhich of the following were found to be true?(A) Both aerobes and anaerobes provide sources of energy through thedecomposition of organic material.(B) Most compounds containing carbon are unavailable as energy sources exceptto some bacteria and fungi.(C) Bacteria and fungi break down inert material in ways that do not involveoxidation.(D) Many compounds remain inert, even in the presence of bacteria and fungi.(E) Bacteria and fungi assist in the synthesis of many organic compounds.No. 2-2SECTION AEven as the number of females processed through juvenile courts climbs steadily, an implicit consensus remains among scholars in criminal justice that male adolescents define the delinquency problem in the United States. We suggest two reasons why this view persists. First, female adolescents are accused primarily of victimless crimes, such as truancy, that do not involve clear-cut damage to persons or property. If committed by adults, these actions are not even considered prosecutable; if committed by juvenile males, they have traditionally been looked on leniently by the courts. Thus, ironically, the plight of female delinquents receives little scrutiny because they are accused of committing relatively minor offenses. Second, the courts have long justified so-called preventive intervention into the lives of young females viewed as antisocial with the rationale that women are especially vulnerable. Traditional stereotypes of women as the weaker and more dependent sex have led to earlier intervention and longer periods of misdirected supervision for female delinquents than for males.17. Which of the following statements best expresses the irony pointed out by theauthors in lines 13-16 of the passage?(A) Female delinquents tend to commit victimless crimes more frequently thantheir male counterparts.(B) The predicament of male delinquents receives more attention than that offemales because males are accused of more serious crimes.(C) Adults are frequently punished less severely than adolescents for committingmore serious crimes.(D) The juvenile justice system cannot correct its biases because it does not evenrecognize them.(E) Although the number of female delinquents is steadily increasing, the crimesof which they are accused are not particularly serious.18. It can be inferred from the passage that the authors believe traditional stereotypesof women to be(A) frequently challenged(B) persistently inexplicable(C) potentially harmful(D) rapidly changing(E) habitually disregarded19. The passage suggests that scholars in criminal justice could be criticized forwhich of the following?(A) Underestimating the seriousness of juvenile crime(B) Rationalizing the distinction made between juveniles and adults in the legalsystem(C) Concerning themselves too little with the prevention of juvenile delinquency(D) Focusing on those whose crimes have involved damage to persons orproperty(E) Failing to point out injustices in the correctional systemScattered around the globe are more than one hundred regions of volcanic activity known as hot spots. Unlike most volcanoes, hot spots are rarely found along the boundaries of the continental and oceanic plates that comprise the Earth’s crust; most hot spots lie deep in the interior of plates and are anchored deep in the layers of the Earth’s surface. Hot spots are also distinguished from other volcanoes by their lavas, which contain greater amounts of alkali metals than do those from volcanoes at plate margins.In some cases, plates moving past hot spots have left trails of extinct volcanoes in much the same way that wind passing over a chimney carries off puffs of smoke. It appears that the Hawaiian Islands were created in such a manner by a single source of lava, welling up from a hot spot, over which the Pacific Ocean plate passed on a course roughly from the east toward the northwest, carrying off a line of volcanoes of increasing age. Two other Pacific island chains—the Austral Ridge and the Tuamotu Ridge—parallel the configuration of the Hawaiian chain; they are also aligned from the east toward the northwest, with the most recent volcanic activity near their eastern terminuses.That the Pacific plate and the other plates are moving is now beyond dispute; the relative motion of the plates has been reconstructed in detail. However, the relative motion of the plates with respect to the Earth’s interior cannot be determined easily. Hot spots provide the measuring instruments for resolving the question of whether two continental plates are moving in opposite directions or whether one is stationary and the other is drifting away from it. Themost compelling evidence that a continental plate is stationary is that, at some hot spots, lavas of several ages are superposed instead of being spread out in chronological sequence. Of course, reconstruction of plate motion from the tracks of hot-spot volcanoes assumes that hot spots are immobile, or nearly so. Several studies support such an assumption, including one that has shown that prominent hot spots throughout the world seem not to have moved during the past ten million years.Beyond acting as frames of reference, hot spots apparently influence the geophysical processes that propel the plates across the globe. When a continental plate comes to rest over a hot spot, material welling up from deeper layers forms a broad dome that, as it grows, develops deep fissures. In some instances, the continental plate may rupture entirely along some of the fissures so that the hot spot initiates the formation of a new ocean. Thus, just as earlier theories have explained the mobility of the continental plates, so hot-spot activity may suggest a theory to explain their mutability.20. The primary purpose of the passage is to(A) describe the way in which hot spots influence the extinction of volcanoes(B) describe and explain the formation of the oceans and continents(C) explain how to estimate the age of lava flows from extinct volcanoes(D) describe hot spots and explain how they appear to influence and record themotion of plates(E) describe the formation and orientation of island chains in the Pacific Ocean21. According to the passage, hot spots differ from most volcanoes in that hot spots(A) can only be found near islands(B) are active whereas all other volcanoes are extinct(C) are situated closer to the earth’s surface(D) can be found along the edges of the plates(E) have greater amounts of alkali metals in their lavas22. It can be inferred from the passage that evidence for the apparent course of thePacific plate has been provided by the(A) contours of the continents(B) dimensions of ocean hot spots(C) concurrent movement of two hot spots(D) pattern of fissures in the ocean floor(E) configurations of several mid-ocean island chains23. It can be inferred from the passage that the spreading out of lavas of differentages at hot spots indicates that a(A) hot spot is active(B) continental plate has moved(C) continental rupture is imminent。

相关文档
最新文档